Sie sind auf Seite 1von 40

https://t.me/IAS201819 https://t.me/PDF4Exams https://t.

me/PDF4Exams

www.nextias.com

Saket Centre : 316/274, Westend Marg, Saidulajab, Near Saket Metro Station, Delhi-30
Old Rajinder Nagar Centre : Ground Floor, 6, Old Rajinder Nagar (Near Salwan School Gate No.-2) New Delhi-60
Ph: 011-45124642, 8800338066  |  E-mail : info@nextias.com

CSE (Prelims) Test Series-2019


General Studies  |  Test-3

C
Booklet
Time Allowed : 2 hrs.
Maximum Marks: 200
Test Code: 190903

INSTRUCTIONS
Please read each of the following instructions carefully before attempting questions:
1. IMMEDIATELY AFTER THE COMMENCEMENT OF THE EXAMINATION, YOU SHOULD CHECK THAT THIS
TEST BOOKLET DOES NOT HAVE ANY UNPRINTED OR TORN OR MISSING PAGES OR ITEMS, ETC. IF SO,
GET IT REPLACED BY A COMPLETE TEST BOOKLET.
2. Please note that it is the candidate’s responsibility to encode and fill in the Roll Number and Test Booklet
Series A, B, C or D carefully and without any omission or discrepancy at the appropriate places in the OMR
Answer Sheet. Any omission/discrepancy will render the Answer Sheet liable for rejection.
3. You have to enter your Roll Number on the OMR Answer Sheet in the box provided.
4. This Test Booklet contains 100 items (Questions). Each item is printed in English only. Each item comprises
four responses (Answers). You will select the response which you want to mark on the Answer Sheet. In case
you feel that there is more than one correct response, mark the response which you consider the best. In any
case, choose ONLY ONE response for each item.
5. You have to mark all your responses ONLY on the separate answer sheet provided. See directions in the
Answer Sheet.
6. All items carry equal marks.
7. Before you proceed to mark in the Answer Sheet the response to various items in the Test Booklet, you have
to fill in some particulars in the Answer Sheet as per instructions sent to you with your Admission Certificate.
8. After you have completed filling in all your responses on the Answer Sheet and the examination has concluded,
you should hand over to the invigilator only the Answer Sheet. You are permitted to take away with you the Test
Booklet.
9. Sheets for rough work are appended in the Test Booklet at the end.
10. Penalty for wrong answers:
THERE WILL BE PENALTY FOR WRONG ANSWERS MARKED BY A CANDIDATE IN THE OBJECTIVE TYPE
QUESTION PAPERS.
(i) There are four alternatives for the answer to every question. For each question for which a wrong answer
has been given by the candidate, one-third of the marks assigned to that question will be deducted as
penalty.
(ii) If a candidate gives more than one answer, it will be treated as a wrong answer even if one of the given
answers happens to be correct and there will be same penalty as above to that question.
(iii) If question is left blank, i.e., no answer is given by the candidate, there will be no penalty for that question.
11. CHALLENGE THE QUESTION: If students feel that either the question(s)/answer(s) needs to be modified or
require clarification, they can email at feedback@nextias.com by 5 pm, 15.10.2018.

DO NOT OPEN THIS BOOKLET UNTIL YOU ARE ASKED TO DO SO


For any queries kindly email at: feedback@nextias.com
https://t.me/TheHindu_Zone_official
https://t.me/IAS201819 https://t.me/PDF4Exams https://t.me/PDF4Exams

2 | CSE (Prelims) Test Series-2019

Objective Questions

Q.1 He won immortal fame by his patriotism, Which of the statements given above is/are
determined fighting and skillful guerrilla not correct?
operations. In November 1857, he had taken (a) 1 only (b) 2 and 3 only
command of the rebel forces of the state of (c) 3 only (d) 1, 2 and 3
Gwalior and driven Gen. C.A. Windham into Q.6 Which of the following newspapers/journals
his entrenchments at Kanpur. He was defeated was/were related to Dada Bhai Naoroji?
by Sir Colin Campbell. The description refers 1. Voice of India
to: 2. Tribune
(a) Nana Sahib 3. Bengalee
(b) Tantia Tope Select the correct answer using the code
(c) Bakht Khan below:
(d) Azimullah (a) 1 and 2 only (b) 1 only
(c) 3 only (d) 1, 2 and 3
Q.2 Which of the following was/were not among
the rebel leaders of the Revolt of 1857? Q.7 He was a teacher, journalist, writer and
1. Scindhia of Gwalior librarian. He started as a supporter of Brahmo
2. Babu Amar Singh of Jagdishpur Samaj, turned to Vedanta and ended up as an
3. Nizam of Hyderabad upholder of the Vaishnava philosophy of Sri
Chaitanya. Paridarsak, New India and Bande
Select the correct answer using the code given Mataram are some of the journals started by
below: him. The description belongs to:
(a) 2 only (b) 2 and 3 only (a) Bipin Chandra pal
(c) 1 and 3 only (d) 1, 2 and 3 (b) Sri Arbindo Ghosh
(c) Ashwini kumar Dutta
Q.3 The term ‘mofussil ‘ was frequently used (d) W.C Banerjee
in context of history of modern India. It is
related to Q.8 Consider the following pairs of movements
and their founder leaders.
(a) Taxation
Movements Leader
(b) Rural area
1. The Depressed Class : C.N Mudliar
(c) Village headman Mission Society
(d) Temple architecture 2. Self-Respect : Sri Narayan Guru
Q.4 Arrange the following civil and tribal Movements
uprisings in a chronological order: 3. Justice Party : V.R Shinde
1. Revolt of Poligars of Tamil Nadu 4. Aravipuram : Ramaswamy
Movement Naicker
2. Diwan Velu Thampi of Travancore
3. Kittur Uprising by Chennamma Which among the above pairs is/are not
4. Santhal Uprising correctly matched ?
(a) 1 and 2 only (b) 2 and 4 only
Select the correct answer using the code (c) 1, 3 and 4 only (d) 1, 2, 3 and 4
below:
Q.9 Consider the following mutiny which occured
(a) 1-2-3-4 (b) 4-2-3-1
over the course of Indian history.
(c) 3-1-2-4 (d) 2-1-4-3
1. Mutiny by 19th Bengal Native Infantry
Q.5 With reference to Ulgulan rebellion, consider regiment
the following statements: 2. Mutiny by 47th Regiment of Sepoys at
1. It was led by tribal leaders Siddhu and Barrackpore
Kanhu during 1899-1900 3. Vellore Mutiny
2. The origin of dislocation of the locals at the 4. Royal Indian Navy Mutiny
hands of the British and their contractors What is the correct chronological order of the
gave birth to this Rebellion. above events?
3. It largely remained a non-violent (a) 3-2-1-4 (b) 3-1-2-4
movement. (c) 2-3-4-1 (d) 1-4-2-3

© Copyright:   www.nextias.com

https://t.me/TheHindu_Zone_official
https://t.me/IAS201819 https://t.me/PDF4Exams https://t.me/PDF4Exams

Modern History of India, Revision of Test-2 & Current Affairs_190903 | 3

Q.10 The Indian Council Act 1861 is important in Q.14 He condemned the caste system, idolatry
the constitutional history of India for several and superstitions and urged the people to
reasons. Which among the following can be imbibe the spirit of liberty, equality and free-
said to be among those reasons? thinking. The remarks :
1. It laid the foundations of the policy of
legislative devolution ‘Our religion is in the Kitchen. Our God is in
2. It transformed the Viceroy of India’s cooking-pot …. If this one goes for another
executive council into a cabinet run on the century, every-one of us will be in lunatic
portfolio system asylum’, are also related to him. He was
3. It gave framework to the Government (a) Swami Vivekanand
of India whose certain aspects has been (b) Raja Ram Mohan Roy
retained up till now. (c) Mahatma Gandhi
(d) Rabindra Nath Tagore
Select the correct answer using the code given
below. Q.15 With reference to the Subsidiary Alliance as
(a) 1, 2 and 3 (b) 2 and 3 only devised by Lord Wellesley in 1798, consider
(c) 1 and 3 only (d) 1 and 2 only the following statements:
1. Indian rulers were not allowed to have
Q.11 Who among the following started the their independent armed forces and
newspaper? a British armed contingent would be
1. Rast Goftar : Frontier Gandhi stationed in the Territory of the ally.
(Abdul Gaffar Khan) 2. If an Indian ruler died without a male heir,
2. Al-Hilal : Maulana Abul Kalam his kingdom would become part of the
Azad
Company territory
3. Udbodhana : Girish Chandra Ghosh
3. Nawab of Awadh was forced to give over
4. Hindu Patriot : Swami Vivekananda
half of his territory to the Company under
5. Shome Prakash : Dayanada Sarswati
Subsidiary Alliance
Select the correct answer using the code
below: Which of the statements given above is/are
(a) 1, 2 and 3 only (b) 3 and 5 only correct?
(c) 2 only (d) 2, 3, 4 and 5 only (a) 3 only (b) 1 only
(c) 2 and 3 only (d) 1, 2 and 3
Q.12 In India the impact of the very same British
industrialisation led to deindustrialisation Q.16 With reference to Battle of Buxar, consider the
in some sectors. Which of the following following statements:
factors can be attributed to phenomenon of 1. The British forces were led by Robert Clive
deindustrialisation? in this battle
1. India suffered negative globalization price 2. The combined armies of Nawab of Bengal,
shocks in its key manufacturing tradable the Nawab of Awadh and Shah Alam II
2. Excess of exports from India with no fought against the British forces..
equivalent import Which of the statements given above is/are
3. The decline of the Mughal empire correct?
Which of the statements given above is/are (a) 1 only (b) 2 only
correct? (c) Both 1 and 2 (d) Neither 1 nor 2
(a) 1 and 3 only (b) 1 and 2 only
Q.17 Which of the following is/are correct with
(c) 2 only (d) 1, 2 and 3
reference to ‘Succession States’ that arose as a
Q.13 Which of the following pairs is/are matched result of the assertion of autonomy by Mughal
correctly? provinces due to decline of Mughal Empire?
1. The Indian League : P. Mehta 1. Punjab can be categorised as one of the the
2. The Indian Anand Mohan ‘Succession States’
Association Bose 2. Policies of ‘Succession States’ remained
3. Madras Mahajan : Anand Charlu largely secular
Sabha 3. Some of the ‘Succession States’ were
4. Bombay Presidency : Sisir Kumar formed by the former governors of
Association Ghosh provinces under the Mughal Empire.
Select the correct answer using the code given Select the correct answer using the code given
below: below
(a) 1 and 4 only (b) 2 and 3 only (a) 1 and 2 only (b) 2 and 3 only
(c) 1 and 3 only (d) 3 and 4 only (c) 1 and 3 only (d) 1, 2 and 3

© Copyright:   www.nextias.com

https://t.me/TheHindu_Zone_official
Join TelegramGroups
To Boost Your Preparation
PDF4Exams One stop solution for study
Click Here materials of all competitiveexams

The Hindu ZoneOfficial


Newspapers & study Click Here
materials

TestSeries4Exam
All paid test series
Click Here availabblewithoutanycost

Pdfbasket
All e-Magazines
in your hand Click Here
Hindi Books
All study materials
Click Here
in Hindi

eSandesh (An Indian App)

For More download eSandesh App from play store


https://t.me/IAS201819 https://t.me/PDF4Exams https://t.me/PDF4Exams

4 | CSE (Prelims) Test Series-2019


Q.18 Consider the following statements: Which of the statements given above is/are
1. Taccavi was a pilgrimage tax levied on non- correct?
Muslim subjects during Aurangzeb’s rule. (a) 1 only (b) 2 and 3 only
2. Ijarah was a Revenue System where land (c) 1 and 3 only (d) 1, 2 and 3
revenue was collected at fixed rate.
Q.23 Which of the following statements regarding
Which of the statements given above is/are classification of stressed assets by RBI is/are
correct? correct?
(a) 1 only (b) 2 only
1. Special Mention Accounts are those assets
(c) Both 1 and 2 (d) Neither 1 nor 2
on which interest/Principle is overdue for
Q.19 With reference to East India Association, more than 90 days.
consider the following statements: 2. NPA’s on agricultural loans are defined
1. It was established on the intiative of based on crop season.
Dadabhai Naoroji in 1866 in Bombay.
2. Its stated objective was to advocate for and Select the correct answer using the code given
promote public interests and welfare of below:
Indians (a) 1 only (b) 2 only
Which of the statements given above is/are (c) Both 1 and 2 (d) Neither 1 nor 2
correct? Q.24 With reference to Minimum Alternative Tax
(a) 1 only (b) 2 only (MAT), consider the following statements:
(c) Both 1 and 2 (d) Neither 1 nor 2 1. MAT is an attempt to reduce tax avoidance
Q.20 Which of the following can be considered among 2. MAT is applicable to all corporate entities,
the demands of early nationalists of 19th century whether public or private.
in Indian freedom struggle? 3. MAT is not applicable to Special Economic
1. Reduction in expenditure on the army Zone Units
2. Abolition of the salt tax
3. Separation of the executive and the judiciary Which of the statements given above is/are
correct?
Select the correct answer using the code given (a) 1 only (b) 1 and 2 only
below
(c) 2 and 3 only (d) 1, 2 and 3
(a) 1 only (b) 1 and 3 only
(c) 2 and 3 only (d) 1, 2 and 3 Q.25 Which of the following is/are the documents
Q.21 Which of the following statement(s) is/are true presented to the Parliament, besides budget
regarding Indian Councils Act of 1892? speech?
1. Members were given the right to vote on 1. FRBM Act mandated statements
Annual Budget 2. Memorandum explaining the provisions
2. It introduced a principle of representation in in the Finance Bill
the council. 3. Outcome Budget
3. The law member was made a permanent 4. Gender Budget
member.
Select the correct answer using the code given
Select the correct answer using the code given below.
below (a) 1 only (b) 1 and 2 only
(a) 1 only (b) 2 and 3 only (c) 1, 2, and 3 only (d) 1, 2, 3 and 4
(c) 2 only (d) 1, 2 and 3
Q.26 Which of the following statements is/are true
Q.22 With reference to history of census in India,
regarding Integrated Financial Management
consider the following statements
1. The first synchronous census in India was Information System (GIFMIS)?
held in 1881 1. It is administered by Comptroller and
2. ‘Arthashastra’ by ‘Kautilya’ written in the Auditor General of India.
3rd Century BC prescribed the collection of 2. It has been established for more effective
population statistics as a measure of state fiscal management of Government.
policy for taxation
Select the correct answer using the code given
3. During regime of the Mughal king Akbar,
below.
the administrative report ‘Ain-e-Akbari’
included comprehensive data pertaining (a) 1 only (b) 2 only
to population, industry and wealth (c) Both 1 and 2 (d) Neither 1 nor 2

© Copyright:   www.nextias.com

https://t.me/TheHindu_Zone_official
https://t.me/IAS201819 https://t.me/PDF4Exams https://t.me/PDF4Exams

Modern History of India, Revision of Test-2 & Current Affairs_190903 | 5

Q.27 Debt receipts are an important source of Which of the statements given above is/are
financing fiscal deficit. Which of the following not correct?
constitutes Debt Receipts? (a) 1 only (b) 2 only
1. Securities against Small Savings (c) Both 1 and 2 (d) Neither 1 nor 2
2. State Provident Funds Q.32 With reference to liquidity trap situation in an
3. Recoveries of Loans and Advances economy, consider the following statements:
Select the correct answer using the code given 1. A liquidity trap is usually caused by, and
below. in turn perpetuates, deflation.
2. The opportunity cost of holding money
(a) 1 and 2 only (b) 1 and 3 only
becomes zero.
(c) 2 and 3 only (d) 1, 2 and 3
Which of the statements given above is/are
Q.28 Which of the following is/are core of the not correct?
core schemes as per recent classification of (a) 1 only (b) 2 only
schemes? (c) Both 1 and 2 (d) Neither 1 nor 2
1. National Social Assistance Progamme
Q.33 The Washington Consensus is a set of 10
2. Mahatma Gandhi National Rural economic policy prescriptions considered
Employment Guarantee Programme to constitute the “standard” reform package
3. Swachh Bharat Mission promoted for developing countries. Which of
4. Umbrella Scheme for Development of the following is not related to the Washington
Other Vulnerable Groups Consensus?
5. Umbrella Integrated Child Development (a) Moving away from subsidies and
Services (ICDS) administration towards previously
neglected fields with high economic
Select the correct answer using the code given returns
below. (b) Proactive and increased role for the State
(a) 1, 2 and 5 only (b) 1, 2 and 4 only (c) Exchange rates should be managed to
(c) 3, 4 and 5 only (d) 1, 2, 3 and 5 only induce rapid growth in non-traditional
exports
Q.29 With reference to insurance sector in India, (d) Secure Intellectual Property Rights (IPR)
consider the following statements: without excessive costs
1. Insurance density is measured as the
percentage of insurance premium to GDP. Q.34 Financial Sector Assessment Program (FSAP) is a
key instrument of International Monetary Fund
2. Insurance penetration is calculated as the
(IMF) surveillance. With reference to the FSAP,
ratio of premium to population consider the following statements:
Which of the statements given above is/are 1. It is done by the International Monetary
correct? Fund (IMF) alone in advanced economies.
(a) 1 only (b) 2 only 2. Its recommendations are of binding nature
for members of International Monetary
(c) Both 1 and 2 (d) Neither 1 nor 2
Fund (IMF)
Q.30 With reference to the term ‘double coincidence Which of the statements given above is/are
of wants’, consider the following statements: correct?
1. It is a situation when the same commodity (a) 1 only (b) 2 only
is desired by two parties. (c) Both 1 and 2 (d) Neither 1 nor 2
2. It is an essential feature of barter system.
Q.35 With reference to Monetized deficit, consider
Which of the statements given above is/are the following statements:
correct? 1. It is that part of the government deficit
(a) 1 only (b) 2 only which is financed solely by borrowing from
(c) Both 1 and 2 (d) Neither 1 nor 2 the Reserve Bank of India.
2. Monetized deficits are an important
Q.31 With reference to the term ‘Quantitative indicator of the inflationary impact of the
Easing’, consider the following statements: increase in government’s budgetary deficits.
1. It is considered as an unconventional fiscal Which of the statements given above is/are
policy. correct?
2. It generally causes an appreciation in the (a) 1 only (b) 2 only
value of the home country’s currency. (c) Both 1 and 2 (d) Neither 1 nor 2

© Copyright:   www.nextias.com

https://t.me/TheHindu_Zone_official
https://t.me/IAS201819 https://t.me/PDF4Exams https://t.me/PDF4Exams

6 | CSE (Prelims) Test Series-2019


Q.36 With reference to surcharge and cess, consider Select the correct answer using the code given
the following statements: below
1. Cess is levied by the Union Government (a) 1 and 2 only (b) 1 and 3 only
only whereas surcharge may be levied by (c) 2 and 3 only (d) 1, 2 and 3
the Union or state governments
Q.41 Consider the following pairs:
2. The proceeds collected from a surcharge Five Year Plan Major Event
and a cess does not form part of the 1. Fourth Plan : Nationalization of
Consolidated Fund of India 14 major Indian Banks
Which of the statements given above is/are 2. Fifth Plan : Smiling Buddha
correct? underground nuclear
(a) 1 only (b) 2 only Test
(c) Both 1 and 2 (d) Neither 1 nor 2 3. Sixth Plan : Introduction of Indian
National Highway
Q.37 To measure changes in the overall price level System
in an economy, policy makers and economists
monitor a number of different economic Which of the pairs given above is/are correctly
indicators. With reference to the two most matched?
important indicators GDP deflator and the (a) 1 only (b) 1 and 3 only
Consumer Price Index (CPI), consider the (c) 2 and 3 only (d) 1, 2 and 3
following statements Q.42 With reference to Provisioning Coverage Ratio
1. CPI does not include changes in the price (PCR), consider the following statements:
of imported goods 1. It is the ratio of provisioning to gross non-
2. GDP deflator does not account for changes performing assets that indicates the extent
in the price of exported goods of funds RBI has kept aside to cover loan
Which of the statements given above is/are losses.
correct? 2. An increase in the PCR means that provisions
(a) 1 only (b) 2 only have not been made to the extent of the rise
(c) Both 1 and 2 (d) Neither 1 nor 2 in bad loans
Q.38 With reference to Gender Budgeting in India, Which of the statements given above is/are
consider the following statements: correct?
1. A separate budget is demarcated for (a) 1 only (b) 2 only
women in the Union Budget. (c) Both 1 and 2 (d) Neither 1 nor 2
2. All central Ministries/Departments are Q.43 Which of the following sectors are included
required to set up Gender Budgeting Cells. under the Priority Sector Lending?
Which of the statements given above is/are 1. Renewable Energy
correct? 2. Social Infrastructure
(a) 1 only (b) 2 only 3. Export Credit
(c) Both 1 and 2 (d) Neither 1 nor 2 Select the correct answer using the code given
Q.39 With reference to the Index of Industrial below
Production(IIP), consider the following (a) 1 only (b) 1 and 2 only
statements: (c) 2 and 3 only (d) 1, 2 and 3
1. IIP is a composite indicator that measures Q.44 New series of currency notes were launched
long-term changes in the volume of after the demonetization. With reference to the
production of a basket of industrial new series, consider the following statements:
products. 1. The theme of the new series notes is India’s
2. It is compiled and published by Central heritage sites.
Statistics Office. 2. Numerals in devnagari and the logo of
Which of the statements given above is/are Swachh Bharat are new other elements
not correct? added in these notes.
(a) 1 only (b) 2 only 3. The new currency notes have optical fibres
(c) Both 1 and 2 (d) Neither 1 nor 2 which can be seen when the notes are
exposed to ultra-violet lamp.
Q.40 Which of the following indices is/are
published by NITI Aayog Which of the statements given above is/are
1. Composite Water Management Index correct?
2. School Education Quality Index (a) 1 only (b) 1 and 2 only
3. Performance on Health Outcomes index (c) 2 and 3 only (d) 1, 2 and 3

© Copyright:   www.nextias.com

https://t.me/TheHindu_Zone_official
https://t.me/IAS201819 https://t.me/PDF4Exams https://t.me/PDF4Exams

Modern History of India, Revision of Test-2 & Current Affairs_190903 | 7

Q.45 With reference to the acquisition of subsidiary Which of the statements given above is/are
banks of State Bank of India, consider the not correct?
following statements: (a) 1 and 3 only (b) 3 only
1. It is in pursuance of the Indradhanush action (c) 1 and 2 only (d) 1, 2 and 3
plan of the Government.
2. The merger will lead to effective Q.50 With reference to “Banks Board Bureau”,
management of high value credit exposures consider the following statements:
1. It is an recommendatory body under
Which of the statements given above is/are
correct? Reserve Bank of India
(a) 1 only (b) 2 only 2. It recommends for selection of heads of
(c) Both 1 and 2 (d) Neither 1 nor 2 public sector banks.
3. All the members and Chairman of Banks
Q.46 Which of the following can be the implications of Board Bureau are part time.
being declared as fugitive economic offender?
1. His property resulting from the proceeds of Which of the statements given above is/are
crime can be confiscated correct?.
2. His properties abroad are also liable for (a) 1 and 3 only (b) 2 only
confiscation (c) 2 and 3 only (d) 1, 2 and 3
3. He would be disentitled from defending any
civil claim. Q.51 With reference to opportunity cost of a decision
‘X’, consider the following statements:
Select the correct answer using the code given
1. The opportunity cost of ‘X’ is only the
below:
(a) 1 only (b) 1 and 2 only next-best alternative foregone and not any
(c) 2 and 3 only (d) 1, 2 and 3 other alternative.
2. Opportunity cost of ‘X’ is different for each
Q.47 Champion Services Sectors refers to the 12 individual and nation”
identified sectors where the Government wants
to give focused attention for promoting their Which of the statements given above is/are
development, and realizing their potential. Which correct?
of the following are among the champion service (a) 1 only (b) 2 only
sectors? (c) Both 1 and 2 (d) Neither 1 nor 2
1. Accounting and Finance Services
2. Audio Visual Services Q.52 Firms are said to be in perfect competition
3. Education Services when which of the following conditions occur?
1. Many firms are producing identical
Select the correct answer using the code given
below: products.
(a) 3 only (b) 1 and 2 only 2. Sellers and buyers have all relevant
(c) 2 and 3 only (d) 1, 2 and 3 information to make rational decisions
about the product being bought and sold.
Q.48 With reference to Initial Coin Offerings (ICO),
3. Firms can enter and leave the market
consider the following statements:
1. An Initial Coin Offering (ICO) is the without any restrictions.
cryptocurrency space’s rough equivalent to Which of the statements given above is/are
an IPO in the mainstream investment world correct?
2. ICOs are regulated by SEBI in India. (a) 1 and 3 only (b) 2 and 3 only
Which of the statements given above is/are (c) 1 and 2 only (d) 1, 2 and 3
correct?
Q.53 Which of the following conditions can be said
(a) 1 only (b) 2 only
(c) Both 1 and 2 (d) Neither 1 nor 2 to be conducive to Low-Level Equilibrium
Trap?
Q.49 With reference to International Monetary Fund 1. A high correlation between the level of per-
(IMF), consider the following statements: capita income and the rate of population
1. All the members of United Nations are
growth
members of International Monetary Fund
(IMF) 2. Scarcity of uncultivated arable land
2. Each member has equal voting power in 3. Inefficient production methods”
IMF decisions Select the correct answer using the code given
3. International Monetary Fund (IMF) also below.
provides concessional loans to help fight (a) 1 and 3 only (b) 1, 2 and 3
poverty in developing countries
(c) 1 and 2 only (d) 2 and 3 only

© Copyright:   www.nextias.com

https://t.me/TheHindu_Zone_official
https://t.me/IAS201819 https://t.me/PDF4Exams https://t.me/PDF4Exams

8 | CSE (Prelims) Test Series-2019


Q.54 With reference to the Indian National Trust Which of the statements given above is/are
for Art and Cultural Heritage (INTACH), correct?
consider the following statements: (a) 1 only (b) 2 only
1. It has been awarded special consultative (c) Both 1 and 2 (d) Neither 1 nor 2
status with United Nations Economic &
Social Council in August,2018. Q.59 Project Navlekha which aims to enable Indian
2. It is a non profit charitable organisation publishers of regional languages to take their
taking up conservations and protection content online is an initiative of:
efforts for both natural and cultural (a) Google
heritages of India. (b) Microsoft
(c) Indian National Trust for Art and Cultural
Which of the statements given above is/are Heritage
correct?
(d) Ministry of Electronics and Information
(a) 1 only (b) 2 only
Technology
(c) Both 1 and 2 (d) Neither 1 nor 2
Q.60 Which of the following statement is correct
Q.55 Which of the following statements is/are
correct with respect to BIMSTEC? with respect to “ Scrub Typhus”?
1. This regional organization came into (a) It is an invasive species detected near
being following the Bangkok Declaration Andman and Nicobar island.
of 1997. (b) It is a ransomware cyber attack.
2. Founding members of the group are India, (c) Scrub typhus is an infectious disease with
Bhutan, Bangladesh and Nepal. symptoms similar to any viral fever.
3. The Asian Infrastructure Investment Bank (d) It is a newly discovered plant species in
is BIMSTEC’s development partner for the Sundraban mangrove forest.
improving transport infrastructure Q.61 Which of the following is correct with respect
Select the correct answer using the code given to “Pitch to move” initiative?
below: (a) It is a mobility pitch competition for
(a) 1 only (b) 1 and 3 only budding startups in the mobility field.
(c) 2 and 3 only (d) 1, 2 and 3 (b) It is a component of Accessible India
Q.56 Which of the following statements with respect Campaign to facilitate seamless movement
to “Parivesh” portal is/are correct? of the differently abled persons.
1. It is a Single-Window Integrated (c) It is an initiative of environment ministry
Environmental Management System for developing eco bridges to connect
2. It is initiated by Niti Aayog. fragmented wildlife habitats.
(d) It is an initiative of NABARD for the
Select the correct answer using the code given
development of tribal area.
below:
(a) 1 only (b) 2 only Q.62 Which of the following statements with respect
(c) Both 1 and 2 (d) Neither 1 nor 2 to the “National Policy on Biofuels 2018” is/
Q.57 Consider the following statements: are not correct?
1. Devaluation reduces the cost of a country’s 1. It envisages a target of 50% blending of
exports, rendering them less competitive ethanol in petrol by 2030.
in the global market 2. It will help in improving infrastructural
2. Devaluation increases the cost of imports Investments in Rural Areas
strengthening domestic businesses. Select the correct answer using the code given
Which of the statements given above is/are below:
correct? (a) 1 only (b) 2 only
(a) 1 only (b) 2 only (c) Both 1 and 2 (d) Neither 1 nor 2
(c) Both 1 and 2 (d) Neither 1 nor 2
Q.63 Which of the following statements with respect
Q.58 With reference to to the International Council to millets is/are correct?
on Monuments and Sites (ICOMOS), consider 1. These are photo-insensitive crops.
the following statements: 2. Millets are hardy, resilient crops that have
1. It was constituted by UNESCO in 1965 a low carbon and water footprint.
for conservation of architectural and 3. Minimum Support Price for millets has been
archaeological heritage across the world. incresed 50% more than the production cost
2. It advises UNESCO for the implementation by the government.
of the World Heritage Convention.

© Copyright:   www.nextias.com

https://t.me/TheHindu_Zone_official
https://t.me/IAS201819 https://t.me/PDF4Exams https://t.me/PDF4Exams

Modern History of India, Revision of Test-2 & Current Affairs_190903 | 9

Select the correct answer using the code given Q.68 Which of the statements given below is/are
below: correct with respect to the Bombay Natural
(a) 1 and 2 only (b) 1, 2 and 3 History Society:
(c) 2 and 3 only (d) 1 and 3 only 1. BNHS is the partner of World Wildlife
Q.64 Which of the following statements is correct Fund India.
with respect to the NASA’s Space mission 2. ArtificiaI intelligence based image
(Transiting Exoplanet Survey Satellite)TESS? recognition platform “Internet of Birds’’
1. Its pupose is to discover more planets has been developed by BNHS and Deloite
beyond our solar system. to identify birds endemic to India.
2. It will be first-ever spaceborne all-sky transit Select the correct answer using the code given
survey. below:
3. It will be using Transit Photometry method (a) 1 only (b) 2 only
that measures the light curve of distant stars (c) Both 1 and 2 (d) Neither 1 nor 2
for periodic dips in brightness.
Q.69 With reference to Public Finance Management
Select the correct answer using the code given
system (PFMS), consider the following
below:
statements:
(a) 1 and 3 only (b) 2 and 3 only
1. It is mandatory to use PFMS for all Central
(c) 1, 2 and 3 (d) 1 and 2 only
Sector Schemes.
Q.65 Consider the following statements: 2. PFMS is integrated with the Core Banking
1. Ministry of New and Renewable Energy has System (CBS) of only the Public Sector
launched an initiative RUCO (Repurpose Banks.
Used Cooking Oil), to enable conversion of
used cooking oil to bio-diesel Which of the statements given above is/are
2. Reheating of oil leads to increase in the level correct?
of Total Polar Compounds. (a) 1 only (b) 2 only
(c) Both 1 and 2 (d) Neither 1 nor 2
Which of the statements given above is/are
correct? Q.70 With reference to new “Policy Framework for
(a) 1 only (b) 2 only Exploration and Exploitation of Unconventional
(c) Both 1 and 2 (d) Neither 1 nor 2 Hydrocarbons”, consider the following
statements:
Q.66 Which of the following statements is/ are correct
1. CBM contractors are not allowed to exploit
with respect to World Congress of Philosophy?
any other hydrocarbon except CBM in the
1. It is for the first time the event is being hosted
alloted area.
by an Asian nation..
2. The policy is a shift from ‘Uniform
2. Ministry of Culture represents India at the
World congress of philosophy. Licensing Policy’ to ‘One hydrocarbon
Resource Type’ for realizing efficient
Select the correct answer using the code given production.
below:
(a) 1 only (b) 2 only Which of the statements given above is/are
(c) Both 1 and 2 (d) Neither 1 nor 2 not correct?
(a) 1 only (b) 2 only
Q.67 With reference to the Global Liveability Index, (c) Both 1 and 2 (d) Neither 1 nor 2
consider the following statements:
1. It is released by UN Sustainable Q.71 Which of the following belong to the group of
Development Solutions Network to asses islands of Andman and Nicobar ?
the quality of urban cities. 1. Havelock Island
2. The index assigns cities scores on four broad 2. Neil Island
parameters institutional, social, physical and 3. Kiltan Island
4. Baratang Island
economic.
Select the correct answer using the code given
Which of the statements given above is/are
below:
correct?
(a) 1 and 2 only (b) 1, 2 and 3 only
(a) 1 only (b) 2 only
(c) 2 and 3 only (d) 1, 2 and 4 only
(c) Both 1 and 2 (d) Neither 1 nor 2

© Copyright:   www.nextias.com

https://t.me/TheHindu_Zone_official
https://t.me/IAS201819 https://t.me/PDF4Exams https://t.me/PDF4Exams

10 | CSE (Prelims) Test Series-2019


Q.72 Consider the following statements: Which of the statements given above is/are
1. Restriced Areas are areas meant for high correct?
degree of ecologically conservation. (a) 1 only (b) 3 only
2. A foreigner is not normally allowed to (c) 2 and 3 (d) 1, 2 and 3
visit a Protected / Restricted Area . Q.76 With reference to World customs Organisation
Which of the statements given above is/are (WCO), consider the following statements:
correct?? 1. WCO represents Customs administrations
(a) 1 only (b) 2 only across the globe that collectively process
(c) Both 1 and 2 (d) Neither 1 nor 2 world trade
2. It is a specialized agency under the United
Q.73 With reference to recently launched ‘cVIGIL’ Nations system
app, choose the correct statement from the 3. India became the Vice Chair of the
following: Asia Pacific Region of World Customs
(a) It is launched by the Election Commission Organisation for a period of three years in
of India to enable citizens to report July 2018.
violation of code of condut during the
elections.. Which of the statements given above is/are
(b) It is launched by the Central Vigilance correct?
(a) 1 only (b) 1 and 2 only
Commitee to enhance accountability of the
(c) 1, 2 and 3 (d) 2 and 3 only
civil servants.
(c) It is launched by the Ministry of Q.77 Which of the following statements is/are true
Parlimanetary Affairs to facilitate about Swachh Survekshan Grameen 2018?
reporting of funds disbursed under the 1. It is a comprehensive survey for rural India
MPLADS. to provide national ranking of all districts
(d) It is launched by the Ministry of Home of India on the basis of quantitative
Affairs to foster participation of citizens in and qualitative sanitation (Swachhata)
border management. parameters
2. It fosters spirit of cooperative federalism.
Q.74 Which of the following statements with respect 3. It is launched by NITI Aayog.
to Asia-Pacific Institute for Broadcasting
Development (AIBD) is/are correct? Select the correct answer using the code given
1. It is a regional inter-governmental below.
organisation servicing countries of the (a) 2 only (b) 1 and 2 only
United Nations Economic and Social (c) 1, 2 and 3 (d) 2 and 3 only
Commission for Asia and the Pacific (UN- Q.78 The terms “Additional Surveillance Measures
ESCAP) in the field of electronic media (ASM)” and “Graded Surveillance Measure
development. (GSM)” had been in news recently. They
2. Full membership of the organisation is are concerned with the working of which
limited to only the sovereign states. organization ?
(a) Reserve Bank of India
Select the correct answer using the code given
(b) Securities and Exchange Board of India
below:
(c) Central Vigilance Commission
(a) 1 only (b) 2 only
(d) Intelligence Bureau
(c) Both 1 and 2 (d) Neither 1 nor 2
Q.79 Regional Rural Banks (RRBs) were set up with
Q.75 United States has designated India as Strategic the objective to provide credit to the small and
Trade Authorization-1 (STA-1) country. With marginalized sections of the society. Which of
reference to STA-1, consider the following the following agencies have an ownership in
statements: the RRBs?
1. India has become the first Asian country 1. Government of India
to get the Strategic Trade Authorisation-1 2. State Government
(STA-1) status 3. NABARD
2. Only members of the four multilateral 4. Sponsor Banks
export control regimes are eligible for
STA-1 status Select the correct answer using the code given
3. It paved the way for high-technology below:
product sales to India, particularly in civil (a) 2 and 3 only (b) 2 and 4 Only
(c) 1, 2 and 4 only (d) 1 and 3 only
space and defence sectors.

© Copyright:   www.nextias.com

https://t.me/TheHindu_Zone_official
https://t.me/IAS201819 https://t.me/PDF4Exams https://t.me/PDF4Exams

Modern History of India, Revision of Test-2 & Current Affairs_190903 | 11

Q.80 With reference to Central Adoption Resource Q.84 The swadeshi influence as part of swadeshi
Authority(CARA), consider the following movement could be seen in folk music and
statements: art. Which of the following statement(s) is/are
1. CARA is a statutory body of Ministry correct regarding Bengali art and music
of Women and Child Development, 1. Rabindranath Tagore composed Amar
Sonar Bangla during this period.
Government of India.
2. Abanindranath Tagore broke domination of
2. It functions as the nodal body for adoption Victorian Naturalism over Indian art.
of Indian children and is mandated to (a) 1 only (b) 2 only
monitor and regulate in-country and inter- (c) Both 1 and 2 (d) Neither 1 nor 2
country adoptions.
Q.85 Which of the following were introduced by Tipu
3. CARA is designated as the Central
Sultan?
Authority to deal with inter-country 1. new coinage system
adoptions in accordance with the 2. new calendar system
provisions of the Hague Convention on 3. new standards of weight and measurement.
Inter-country Adoption, 1993. 4. ‘Zij Muhammadshahi’ to enable people to
make astronomical observations
Which of the statements given above is/are
correct? Select the correct answer using the code given
(a) 1 and 2 only (b) 1 only below:
(c) 1, 2 and 3 (d) 2 and 3 only (a) 1 and 2 only (b) 1, 2 and 3 only
(c) 2, 3 and 4 only (d) 1, 2, 3 and 4
Q.81 With reference to the Vikas engine, consider
the following statements: Q.86 He neglected administration of the empire. He
himself indulged in court intrigues. Nadir Shah
1. Vikas is a family of liquid fuelled rocket
invaded India during his reign. The next Mughal
engines that powers India’s launch Emperors after him, Ahmad Shah (1748-54) and
vehicles PSLV and GSLV Alamgir-II (1754-59) were too weak to check the
2. Gas Turbine Research Establishment rot that had set in.
(GTRE) has recently improved the thrust
The above description refers to which of the
of the Vikas engine which is expected to
following Mughal ruler?
boost the rocket engine. (a) Bahadur Shah-I
Which of the statements given above is/are (b) Muhammad Shah
correct? (c) Shah Alam-II
(a) 1 only (b) 2 only (d) Aurangzeb
(c) Both 1 and 2 (d) Neither 1 nor 2 Q.87 During the 200 years of the British rule, more
than 40 major and minor famines took place and
Q.82 “Global Slavery Index” is released by which of millions of people died. Which of the following
the following organization? can be considered as the causes of famine
(a) Amnesty International during this period?
(b) Walk Free Foundation 1. Commercialization of agriculture.
(c) The Office of UN Commissioner for 2. Export of food materials.
Human Rights 3. Hoarding and black marketing of food
(d) World Justice Project grain.
Q.83 Consider the following statements: Select the correct answer using the code given
1. Capital Adequacy Ratio (CAR) is the below
amount that banks have to maintain in the (a) 1 and 2 only (b) 2 and 3 only
(c) 1 and 3 only (d) 1, 2 and 3
form of their own funds to offset any loss
that banks incur if any account-holders fail Q.88 Which of the following statements regarding The
to repay dues. Caste Disabilities Removal Act, 1850 is correct?
2. Basel norms have guidelines on CAR. (a) It abolished all laws affecting the rights of
3. Basel norms are given by the World Bank. persons converting to another religion
(b) The act abrogated deprivation of rights
Which of the statements given above is/are and interests of a remarried widow in her
correct? deceased husband’s property
(a) 1 only (b) 2 only (c) It made untouchability a punishable offence
(c) 1 and 2 only (d) 1, 2 and 3 (d) It provided reservation of seats in civil
services for lower castes.

© Copyright:   www.nextias.com

https://t.me/TheHindu_Zone_official
https://t.me/IAS201819 https://t.me/PDF4Exams https://t.me/PDF4Exams

12 | CSE (Prelims) Test Series-2019


Q.89 Consider the following statements 2. The Vernacular Press Act excluded
1. Sir Thomas Roe was sent to Jahangir`s English-language publications
court to obtain permission to open factory
at Surat Which of the statements given above is/are
2. Captain William Hawkins got a Royal correct?
firman (decree) to open factory and trade (a) 1 only (b) 2 only
in any part of Mughal Empire (c) Both 1 and 2 (d) Neither 1 nor 2

Which of the statements given above is/are Q.94 Renaissance in India was essentially a matter of
correct? spirit and new developments in religious,social
(a) 1 only (b) 2 only and political life.Which of the following can be
(c) Both 1 and 2 (d) Neither 1 nor 2 said to be true regarding the same?
1. There was an emphasis on the recovery of
Q.90 Raja Ram Mohan Roy has played a vital
the spiritual tradition and heritage of the
role during the Renaissance in India and is
past.
regarded as a ‘Maker of Modern India’.
2. There was establishment of various
Which of the following themes was/were religious sects and groups.
reflected in the works of Raja Rammohan 3. JC Bose,a contemporary of Raja Ram
Roy? Mohan Roy pionered investigation of
1. Abolition of Sati radio and microwaves and contributed to
2. Promotion of Sanskrit education revival of sciences.
3. Widow remarriage
4. Universal humanism Select the correct answer using the code given
below
Select the correct answer using the code (a) 1 only (b) 1 and 2 only
below: (c) 2 and 3 only (d) 1, 2 and 3
(a) 1, 2 and 3 only (b) 2, 3 and 4 only
(c) 1, 3 and 4 only (d) 1, 2 and 4 only Q.95 The doctrine of lapse declared that if an Indian
ruler died without a male heir his kingdom
Q.91 With reference to British initiative in transport
would“lapse”. Which of the following
and communication in India, consider the
kingdoms was not annexed using the above
following statements:
1. The first telegraph line opened in 1853 was criteria?
between Calcutta to Bombay (a) Sambalpur
2. The first railway line in India was built by (b) Udaipur
private company in 1833 from Bombay to (c) Nagpur
Thane. (d) Awadh

Which of the statements given above is/are Q.96 Consider the following pairs:
correct? Autonomous Founders
(a) 1 only (b) 2 only Kingdoms
(c) Both 1 and 2 (d) Neither 1 nor 2 1. Hyderabad : Nizam-uI-Mulk
2. Bengal : Murshid Quli Khan
Q.92 Which of the following statements regarding
3. Avadh : Wajid Ali Shah
Factories Act of 1881 is not correct?
(a) The Factories Act of 1881 was enacted at the Which of the pairs given above is/are correctly
instance of British cotton manufacturers matched?
(b) The Factories Act of 1881 was the first (a) 1 only (b) 1 and 2 only
legislation directly affecting industrial (c) 2 and 3 only (d) 1, 2 and 3
labourers in the country
(c) It applied to all manufacturing Q.97 The foreign powers had established various
establishments using power driven factories (trading posts) in India by the eighteenth
machinery century. Arrange the following locations of
(d) It ruled that no children below the age of factories from North to South.
14 years could be employed in a factory 1. Chinsura
2. Masulipatnam
Q.93 With reference to Vernacular Press Act 1878 3. Salsetee
also known as ‘Gagging Act’, consider the
4. Serampore
following statements:
1. It repealed the Press Act of 1835 known Select the correct answer using the code below:
as the most liberal Press Act in Indian (a) 4, 1, 2, 3 (b) 4, 1, 3, 2
history. (c) 1, 4, 2, 3 (d) 1, 4, 3 ,2

© Copyright:   www.nextias.com

https://t.me/TheHindu_Zone_official
https://t.me/IAS201819 https://t.me/PDF4Exams https://t.me/PDF4Exams

Modern History of India, Revision of Test-2 & Current Affairs_190903 | 13

Q.98 “He was one of the earliest religious Which of the pairs given above is/are
reformer in Western India, known popularly correctly matched?
as “Lokahitwadi, who wrote in Marathi, (a) 1 and 2 only (b) 3 only
made powerful rationalist attacks on Hindu (c) 1, 2 and 3 (d) None of the Above
orthodoxy, and preached religious and social
equality. “ Q.100 With reference to Pitts India Act 1784, consider
the following statements.:
The above description describes which 1. It gave the British Government supreme
famous personality during the freedom control over the Company’s affairs and its
struggle. administration in India.
(a) Jyotiba Phule 2. It marked the beginning of the centralizing
(b) Pherozshah Mehta tendency that reached its climax under the
(c) Gopal Hari Deshmukh Charter Act of 1833.
(d) Behramji Malabari 3. It was passed after the First Anglo-Mysore
Q.99 In the nineteenth century, the British devised war but before the Anglo-Maratha war.
various systems to exploit resources generated Which of the statements given above is/are
from land. With reference to popular revolts/ correct?
uprisings and the underlying revenue (a) 1 only
system/ cultivation system responsible for it, (b) 2 only
consider the following pairs (c) 1 and 3 only
1. Pabna Revolt : Ryotwari (d) 1, 2 and 3
2. Indigo Revolt : Zamindari
3. Deccan Uprising : Mahalwari nnnn

© Copyright:   www.nextias.com

https://t.me/TheHindu_Zone_official
https://t.me/IAS201819 https://t.me/PDF4Exams https://t.me/PDF4Exams

14 | CSE (Prelims) Test Series-2019

SPACE FOR ROUGH WORK

© Copyright:   www.nextias.com

https://t.me/TheHindu_Zone_official
https://t.me/IAS201819 https://t.me/PDF4Exams https://t.me/PDF4Exams

Modern History of India, Revision of Test-2 & Current Affairs_190903 | 15

SPACE FOR ROUGH WORK

© Copyright:   www.nextias.com

https://t.me/TheHindu_Zone_official
https://t.me/IAS201819 https://t.me/PDF4Exams https://t.me/PDF4Exams

16 | CSE (Prelims) Test Series-2019

BIG LEARNINGS MADE E ASY

An initiative of Group

17
Books

Modern Indian History


History of Medieval India

History of Ancient India

History of the World

Indian Economy
Internal Security
Indian Society
Post-Independence History
International Relations

Science & Technology


Physical & World Geography
& Disaster Management

Ecology and Environment


Indian Geography

Life Sciences

Indian Polity

Ethics

Corporate Office: 44A/1, Kalu Sarai, New Delhi-110016 | 011-45124660, 08860378007


© MADE
Copyright: at leading book stores across India and on major e-comerce sites like:
EASY books are available   www.nextias.com

https://t.me/TheHindu_Zone_official
https://t.me/IAS201819 https://t.me/PDF4Exams https://t.me/PDF4Exams

Modern History of India, Revision of Test-2 & Current Affairs_190903 | 1

CSE (Prelims) Test Series-2019 | Test-3


Detailed Explanations

1. (b) 3. (b)
As per the given description Tantia Tope is the The term mofussil refers to rural area. During
Correct answer. the British rule, the indigo planters, nearly all
Tantia Tope was in the service of the former Europeans, compelled the tenants to grow indigo
peshwa (ruler) of the Maratha confederacy, which they processed in factories set up in rural
Baji Rao II, and of his adopted son Nana Sahib, (mofussil) areas.
who was also prominent in the mutiny. Tantia 4. (a)
Tope was defeated by Sir Colin Campbell (later
The Poligar Wars: The Poligar Wars were wars
Baron Clyde) on December 6 but remained at
fought between the Poligars (Palaiyakkarars) of
Kalpi, the scene of his defeat. In March 1858 he
the former Tirunelveli Kingdom in Tamil Nadu,
moved to Jhansi, whose rani (queen) Lakshmi
India and the British East India Company forces
Bai was besieged by British forces. His forces between March 1799 to 1805. The British finally
were broken up on June 19, but he continued won after carrying out gruelling protracted
resistance as a guerrilla fighter in the jungle until jungle campaigns against the Poligar armies. The
he was betrayed the following April. He was suppression of the Poligar rebellion resulted in
tried and executed at Shivpur. the liquidation of the influence of the chieftains.
General Bakht Khan led the forces in Delhi. The British assumed direct control over Tamil
Bakht Khan was an ordinary subedar of artillery Nadu. The Poligar system which had flourished
in the British army. for two and a half centuries came to a violent
Azimullah was another loyal servant of Nana end and the company introduced a Zamindari
Sahib. He was an expert in political propaganda. settlement in its place.
Travancore Rebellion: In 1795 a subsidiary
2. (c)
alliance was established between the British East
Statement 1 is Correct: The Sindhia of Gwalior India Company and the Kingdom of Travancore.
provided help to Britishers in suppressing the Under the treaty, the Company was to maintain
revolt. a subsidiary military force in or near Travancore
Statement 2 is not correct: Babu Amar Singh was to defensively aid the kingdom from foreign
a revolutionary in the Indian Rebellion of 1857 powers, and the maintenance costs would be
and brother of Babu Kunwar Singh. After the paid by the government of Travancore. The King
death of Babu Kunwar Singh on 24 April 1858, could not pay the subsidy cost. But the Britishers
Babu Amar Singh became the chief of the army demanded full subsidy. So in 1808 open rebellion
and despite heavy odds, continued the struggle broke out in Travancore.
and for a considerable time ran a parallel The Kittur Rebellion (1824-29): The British
government in the district of Shahabad. prevented the adopted son of the chief of Kittur
Statement 3 is Correct: Nizam of Hyderabad also (in Karnataka) from assuming power by taking
provided active help to the British in suppressing over the administration, that resulted in revolt.
the revolt. The revolt was led by the widow of the chief,
Chennamma.
Additional Information
Santhal Rebellion: Before the British advent in
One of the major weakness of 1857 revolt was that
India, Santhals resided in the hilly districts of
many Indian rulers did not support the rebellion.
Manbhum, Barabhum, Chhotanagpur, Palamau,
They even provided help to the Britishers.
and Birbhum. But as the agents of the new
Among them were the Sindhia of Gwalior, the
colonial rule claimed their rights on the lands,
Holkar of Indore, the Nizam of Hyderabad, the
the Santhals retreated to the hills of Rajmahal.
Raja of Jodhpur and other Rajput rulers, the
After a brief period, the British operatives with
Nawab of Bhopal, the rulers of Patiala, Nabha,
their native underlings i.e. the local landlords lay
Jind, and other Sikh chieftains of Punjab, the
claim on this new land as well. Zamindars and
Maharaja of Kashmir, the Ranas of Nepal, and the money lenders allured them by goods lent
many other ruling chiefs, and a large number of to them on loans, through corrupt practices of
big Zamindars who provided active help to the the money lenders, the loan grew to prohibitive
British in supressing the revolt. proportions, for repaying which entire family

© Copyright:   www.nextias.com

https://t.me/TheHindu_Zone_official
https://t.me/IAS201819 https://t.me/PDF4Exams https://t.me/PDF4Exams

2 | CSE (Prelims) Test Series-2019


had to work as bonded labourers. This under Sisir Kumar Ghosh and Motilal Ghosh,
dispossession turned the Santhals into rebels and Sudharak under G.K. Gokhale, Indian
finally they took an oath to launch an attack on Mirror under N.N. Sen, Voice of India under
the ruling authority, i.e. the British. In 1855, two Dadabhai Naoroji, Hindustani and Advocate
Santal rebel leaders, Sidhu and Kanhu Murmu, under G.P. Varma and Tribune by Sarday Dyal
mobilized ten thousand Santhals and declared a
Singh Majeethia.
rebellion against British colonists. Sidhu Murmu
had accumulated about ten thousands Santhal to 7. (a)
run parallel government against British rule. The above description refers to Bipin Chandra
5. (d) Pal.
Statement 1 is not correct: Ulgulan rebellion was Bipin Chandra Pal is known as the 'Father of
led by famous tribal leader Birsa Munda. Siddhu Revolutionary Thoughts' in India and was one
and Kanhu led the Santhal Rebellion. of the freedom fighters of India. He started his
Statement 2 is not correct: The Ulgulan rebellion career as the Headmaster of a high school in
of the Munda tribesmen occurred during 1899- Cuttack. He also took up journalism, started the
1900. For over thirty years the Munda sardars Bengali weekly Paridarsak, served as Assistant
had been struggling against the destruction of Editor of the Bengal Public Opinion and the
their system of common land holdings by the Lahore Tribute. Bipin Chandra’s religious views
intrusion of jagirdar, thikadar (revenue farmers) underwent a process of evolution. He started as
and merchant moneylenders. This process a Brahmo supporter in his youth, but in his later
of land alienation had begun long before the life he was greatly influenced by the Vedantic
advent of the British. But the establishment and philosophy of Sankaracharya, and finally, he
consolidation of British rule accelerated the was drawn to the Vaishnava philosophy of Sri
mobility of the non-tribal people into the tribal Chaitanya. In addition to Paridarsak, he started a
regions. weekly journal New India, and Bande Mataram.
Statement 3 is not correct: On Christmas Eve,
1899, Birsa proclaimed a rebellion to establish 8. (d)
Munda rule in the land and encouraged ‘the Pair 1 is not correct: The Depressed Class Mission
killing of thikadars and jagirdars and Rajas and Society was started by Vitthal Ramji Shinde in
Hakims (rulers) and Christians.’ He declared 1906.
that ‘there was going to be a fight with the Pair 2 is not correct: Self Respect movements
dikus, the ground would be as red as the red was led by Ramaswamy Naicker (also called as
flag with their blood.’ To bring about liberation, Periyar) in Tamil Nadu against Brahminism. A
Birsa gathered a force of 6,000 Mundas armed number of political parties in Tamil Nadu, such
with swords, spears, battle-axes, and bows and as Dravida Munnetra Kazhagam (DMK) and
arrows.However, the non-tribal poor were not to All India Anna Dravida Munnetra Kazhagam
be attacked.
(AIADMK) owe their origin to the Self-respect
6. (b) movement.
Option 1 is correct: In 1883, Dada Bhai Naoroji Pair 3 is not correct: Jusice party, officially called
started the Voice of India in Bombay and later the South Indian Liberal Federation, was a
incorporated it into the Indian Spectator. political party in the Madras Presidency of British
Option 2 is not correct: Tribune was founded by India. It was established in 1916 in Madras by T.
Sardar Dyal Singh Majeethia, a philanthropist on M. Nair and P. Theagaraya Chetty.
2nd February 1881, in Lahore (now in Pakistan). Pair 4 is not correct: Aravipuram Movement was
Option 3 is not correct: Surendranath Banerjea launched by Sri Narayana Guru on Shivaratri of
started Bengalee in 1879. 1888. On that day, Sri Narayana Guru defied
Additional Information: the religious restrictions traditionally placed
Powerful newspapers emerged during these years on the Ezhava community, and consecrated an
under distinguished and fearless journalists. idol of Shiva at Aravipuram. The Aravipuram
These were the Hindu and Swadesamitran under movement has a great significance in South
the editorship of G. Subramaniya Iyer, Kesari India as it led to the beginning of many other
and Mahratta under B.G. Tilak, Bengalee under socio-religious movements. The temple entry
Surendranath Banerjea, Amrita Bazar Patrika movement is one of them.

© Copyright:   www.nextias.com

https://t.me/TheHindu_Zone_official
https://t.me/IAS201819 https://t.me/PDF4Exams https://t.me/PDF4Exams

Modern History of India, Revision of Test-2 & Current Affairs_190903 | 3

9. (a) and Bombay. Thus, the Act laid the foundations


Sepoy Mutiny at Bengal: In February 1857, the of legislative devolution.
19th Bengal Native Infantry regiment became Statement 2 is correct: Lord Canning, who
concerned that new cartridges they had been was the Governor-General and Viceroy at the
issued were wrapped in paper greased with time, introduced the portfolio system. In this
cow and pig fat, which had to be opened by system, each member was assigned a portfolio
mouth thus affecting their religious sensibilities. of a particular department. For the executive
Mangal Pandey Attacked his officer and was functions of the Council, there were five
later executed. This led to a widespread sepoy members for home, military, law, revenue and
mutiny in Bengal. finance. A sixth member for public works was
47th Regiment of sepoys at Barrackpore: The added in 1874.
Barrackpore mutiny was a rising of native Statement 3 is correct: Many provisions of
Indian sepoys against their British officers in the Act are retained till date as explained with
Barrackpore in 1824. The incident occurred when respect to cabinet system as an example.
the British East India Company was fighting the Additional Information
First Anglo-Burmese War (1824–26). The 47th
Other salient features of the Indian Council Act
Regiment of sepoys at Barrackpore refused to
1861 are:
go to Burma by the sea-route. The Regiment was
disbanded, its unarmed men were fired upon 1. Lord Canning nominated three Indians to the
by artillery, and the leaders of the sepoys were Council in 1862 namely, the Raja of Benares,
hanged. the Maharaja of Patiala and Sir Dinkar Rao.
Vellore mutiny: The Vellore mutiny on 10 July 2. Any bill related to public revenue or debt,
1806 was the first instance of a large-scale and military, religion or foreign affairs could not
violent mutiny by Indian sepoys against the East be passed without the Governor-General’s
India Company. The revolt, which took place in assent.The Viceroy had the power to overrule
the South Indian city of Vellore lasted one full the council if necessary.
day, during which mutineers seized the Vellore 3. The Governor-General also had the power to
Fort and killed or wounded 200 British troops. promulgate ordinances without the council’s
The sepoys at Vellore mutinied were crushed concurrence during emergencies.
with terrible violence. 4. The Secretary of State for India in Britain
Royal Indian Navy Mutiny: The growing could also dissolve any Act passed by the
nationalist sentiment, that reached a crescendo Governor-General’s Council.
around the INA trials, developed into violent
11. (c)
confrontations with authority in the winter of
1945-46. There were three upsurges — one on 21 Pair 1 is not correct: Rast Goftar was written by
November 1945 in Calcutta over the INA trials; Dada Bhai Naoroji in 1851
the second on 11 February 1946 in Calcutta to Pair 2 is correct: Maulana Abul Kalam Azaad
protest against the seven year sentence given propagated his rationalist and nationalist ideas
to an INA officer and the third in Bombay of in his newspaper Al Hilal which he brought out
18 February 1946 when the ratings of the Royal in 1912.
Indian Navy (RIN) went on strike. The upsurges Pair t 3 is not correct: Udbodhana was written by
followed a fairly similar pattern: an initial stage Swami Vivekananda
when a group (such as students or ratings) defied Pair 4 is not correct: Hindu Patriot was written
authority and was repressed, a second stage by Girish Chandra Ghosh
when people in the city joined in, and finally
Pair 5 is not correct: Shome Prakash was written
a third stage when people in other parts of the
by Ishwar Chandra Vidyasagar
country expressed sympathy and solidarity.
12. (d)
10. (a)
Statement 1 is correct: Britishers wanted to
Statement 1 is correct: The Act enlarged the
make India a consumer of its exported products.
Governor-General’s Executive Council for the
They wanted to use India just as a raw material
purpose of making laws. The Governor-General
supplying nation. So indian imports were priced
could now add from six to twelve members
very high in England. This led to Indian goods
to the Executive Council. Moreover the Act
suffering negative globalisation price shocks in
restored the legislative powers of the making
its key manufacturing tradables.
and amending laws to the provinces of Madras

© Copyright:   www.nextias.com

https://t.me/TheHindu_Zone_official
https://t.me/IAS201819 https://t.me/PDF4Exams https://t.me/PDF4Exams

4 | CSE (Prelims) Test Series-2019


Statement 2 is correct: Some of the sectors also the allying Indian State was compelled to accept the
expereinced deindustrialization due to lack of permanent stationing of a British force within his
imports. India’s most important article of export territory and to pay a subsidy for its maintenance.
was cotton textiles. India also exported raw silk The system of Subsidiary Alliances also led to the
and silk fabrics, hardware, indigo, saltpetre, disbandment of the armies of the protected states.
opium, rice, wheat, sugar, pepper and other Statement 2 is not correct: It was under Doctrine
spices, precious stones, and drugs.It exported of Lapse,not under Subsidiary Alliance that If an
more than it imported and its trade was balanced Indian ruler died without a male heir, his kingdom
by import of silver and gold. These items would become part of the Company territory.
seldom went value addition and lacked robust
Statement 3 is correct: The Nawab of Avadh
base for production. Hence although India had
positive balance of trade but still underwent was forced to sign a Subsidiary Treaty in 1801. In
deindustrialization. return for a larger subsidiary force, the Nawab
was made to surrender to the British nearly half
Statement 3 is correct: The decline of nobility
of his kingdom consisting of Rohilkhand and the
left a lot of manufacturing sectors with no
territory lying between the Ganga and the Jamuna.
buyers, for example: armoury. This too led to
Sometimes the ruler ceded part of his territory
deindustrialization.
instead of paying annual subsidy. The Subsidiary
13. (b) Treaty also usually provided that the Indian ruler
Pair 1 is not correct: Indian League was founded would agree to the posting at his court of a British
by Sisir Kumar Ghosh along with Sambhu Resident, that he would not employ any European
Charan Mukherjee in Calcuttta in 1875. in his service without the approval of the British,
and that he would not negotiate with any other
Pair 2 is correct: The Indian National Association,
Indian ruler without consulting the Governor-
also known as Indian Association, was the first
General. In return the British undertook to defend
avowed nationalist organization founded in
British India by Surendranath Banerjee and the ruler from his enemies.
Ananda Mohan Bose in 1876. Additional information:
Pair 3 is correct: Madras Mahajan Sabha: In 1884, Subsidiary Alliance was a system devised by Lord
M. Veeraraghavachariar, G. Subramania Iyer Wellesley in 1798. All those who entered into such
and P. Anandacharlu established the Madras an alliance with the British had to accept certain
Mahajana Sabha. terms and conditions:
Pair 4 is not correct: The Bombay Presidency (a) The British would be responsible for protecting
Association was started by Badruddin Tyabji, their ally from external and internal threats to
Pherozshah Mehta and K.T. Telang in 1885. their power.
(b) In the territory of the ally, a British armed
14. (a)
contingent would be stationed.
Swami Vivekananda (1863 –1902), born
Narendranath Datta, was an Indian Hindu (c) The ally would have to provide the resources
monk, a chief disciple of the 19th-century Indian formaintaining this contingent.
mystic Ramakrishna. He was a key figure in (d) The ally could enter into agreements with
the introduction of the Indian philosophies of other rulers or engage in warfare only with the
Vedanta and Yoga to the Western world and permission of the British.
is credited with raising interfaith awareness,
16. (b)
bringing Hinduism to the status of a major world
religion during the late 19th century. He was a Statement 1 is not correct: Battle of Buxar was
major force in the revival of Hinduism in India, fought in 1764 between the forces under the
and contributed to the concept of nationalism command of the British East India Company, led
in colonial India. Vivekananda founded the by Hector Munro. It was in the Battle of Plassey
Ramakrishna Math and the Ramakrishna that the British East India Company led under
Mission. He is perhaps best known for his speech Robert Clive defeated Siraj-ud-Daulah, Nawab of
at the Parliament of the World's Religions in Bengal in 1757.
Chicago in 1893. Statement 2 is correct: The combined armies of
Mir Qasim, Nawab of Bengal till 1763; the Nawab
15. (*)
of Awadh; and the Mughal Emperor Shah Alam II
Statement 1 is correct: Under the Subsidiary fought against the British in the Battle of Buxar.
Alliance system of Lord Wellesley, the ruler of

© Copyright:   www.nextias.com

https://t.me/TheHindu_Zone_official
https://t.me/IAS201819 https://t.me/PDF4Exams https://t.me/PDF4Exams

Modern History of India, Revision of Test-2 & Current Affairs_190903 | 5

17. (b) 19. (b)


On the debris of the Mughal Empire and its political Statement 1 is not correct: In 1866, Dadabhai
system arose a large number of independent and Naoroji organised the East India Association in
semi-independent powers such as Bengal, Avadh, London to discuss the Indian question and to
Hyderabad, Mysore and the Maratha Kingdom. influence British public men to promote Indian
Some arose as a result of the assertion of autonomy welfare. Later he organised branches of the
by governors of Mughal provinces, others were the Association in prominent Indian cities as well.
product of rebellion against Mughal authority. Statement 2 is correct: The key activities of the
Statement 1 is not correct: Punjab was the product Society were to ventilate the Indian political,
of rebellion against the Mughal authority. Other social and literary subjects and formulate the
Indian grievances.
kingdoms like Maratha, Jat, Afghan too were
product of rebellion by local cheiftains, zamindars 20. (d)
and peasants. Statement 1 is correct: Reduction in the
Statement 2 is correct: The rulers established law expenditure on the army was a demand by the
and order and viable economic and administrative early nationalists.
states. The politics of these states were invariably Statement 2 is correct: Abolition of the salt tax
non-communal or secular, the motivations of their was a demand by the early nationalists.
rulers being similar in economic and political terms. Statement 3 is correct: Separation of the
These rulers did not discriminate on religious executive and the judiciary was a demand by the
grounds in public appointments, civil or military; early nationalists.
nor did the rebels against their authority pay much Additional information:
attention to the religion of the rulers.
Following were the other demands of early
Statement 3 is correct: Indian nationalists:
The state of Hyderabad was founded by Nizam-ul- (a) Improvement in the conditions of work of the
Mulk Asaf Jah in 1724. He was one of the leading plantation labourers.
nobles of the post-Aurangzeb era. He laid the (b) Reduction of land revenue.
foundations of the Hyderabad State which he ruled
(c) Indianization of the higher grades of
with a strong hand. He never openly declared his
administrative services.
independence from the Central Government but
in practice he acted like an independent ruler. He (d) They opposed the curtailment of the powers
waged wars, concluded peace, conferred titles, and of the juries.
gave jagirs and offices without reference to Delhi. (e) Development of agricultural banks to save
the peasant from the clutches of the money-
Murshid Quh Khan and Alivardi Khan, made
lender.
Bengal virtually independent, Even though
Murshid Quh Khan was made Governor of Bengal (f) Extension of medical and health facilities
as late as 1717, he had been its effective ruler since (g) Improvement of the police system.
1700, when he was appointed its Dewan. He soon (h) They opposed the official policy of disarming
freed himself from central control though he sent the people and asked the government to trust
regular tribute to the Emperor. the people and grant them the right to bear
The founder of the autonomous kingdom of arms and thus defend themselves and their
Avadh was Saadat Khan Burhan-ul-Mulk who country in times of need.
was appointed Governor of Avadh in 1722. (i) They laid a great deal of emphasis on the
spread of primary education among the
18. (d) masses. They also demanded greater facilities
Statement 1 is not correct: Taccavi was agricultural for technical and higher education.
loan granted to the poor cultivators to relieve their
21. (c)
distress as well as to enable them to pay land
Statement 1 is not correct: The members were
revenue in time.
given the right to discuss the annual budgets
Statement 2 is not correct: Ijarah was the practice though they could not vote on them as per the
of entitling farming rights to the highest bidder, Indian Councils Act of 1892.
who was permitted to raise what he could from the
Statement 2 is correct: Number of members of
peasantry. The practice was followed during the the Imperial Legislative Council as well as of
Later Mughal era.

© Copyright:   www.nextias.com

https://t.me/TheHindu_Zone_official
https://t.me/IAS201819 https://t.me/PDF4Exams https://t.me/PDF4Exams

6 | CSE (Prelims) Test Series-2019


the provincial councils was increased and some 24. (b)
of these members could be elected indirectly by Statement 1 is correct: Minimum Alternative
Indians, but the official majority remained. Tax (MAT) was introduced to bring into the
Statement 3 is not correct: Charter Act,1833 added tax net “zero tax companies” which in spite
a law member to the council (Lord Macaulay) and of having earned substantial book profits and
a law commission was instituted for codification of having paid big dividends, do not pay any tax
laws. The law member was made a full member by due to various tax concessions and incentives
the Charter Act of 1853. provided under the income tax law. MAT is
levied at the rate of 18.5% plus surcharge and
22. (d) cess as applicable on book profit.
Statement 1 is correct: A systematic and modern Statement 2 is correct: The provisions of MAT
population census, in its present form was are applicable to every company whether
conducted non synchronously between 1865 and public or private and whether Indian or
1872 in different parts of the country. This effort foreign. As per section 115JB of Income Tax
culminating in 1872 has been popularly labeled as Act, every taxpayer being a company is liable
the first population Census of India However, the to pay MAT, if the Incometax (including
first synchronous Census in India was held in 1881. surcharge and cess) payable on the total
Statement 2 is correct: The earliest literature 'Rig- income, computed as per the provisions of the
Veda' reveals that some kind of population count Income-tax Act in respect of any year is less
was maintained during 800-600 BC in India. The than 18.50% of its book-profit + surcharge (SC)
celebrated 'Arthashastra' by 'Kautilya' written in + health & education cess.
the 3rd Century BC prescribed the collection of Statement 3 is not correct: Exemption from
population statistics as a measure of state policy MAT has been withdrawn w.e.f. 1.4.2012 for
for taxation. It contained a detailed description of the special economic zones, hence the units
methods of conducting population, economic and operating in SEZs are liable to pay MAT.
agricultural censuses.
Statement 3 is correct: During the regime of the 25. (c)
Mughal king Akbar, the administrative report 'Ain- Annual Financial statement, Demand for
e-Akbari' included comprehensive data pertaining Grants and Finance Bill are mandated by
to population, industry, wealth and many other Art. 112,113, and 110(a) of the Constitution of
characteristics. India respectively, while some documents are
presented as per the provisions of the Fiscal
23. (b) Responsibility and Budget Management Act
Statement 1 is not correct: The classification of (FRBM), 2003. Other documents are in the
Special Mention Accounts (SMA) was introduced nature of explanatory statements supporting
by the RBI in 2014, to identify those accounts that the mandated documents.
has the potential to become an NPA/Stressed Options 1, 2 and 3 are correct: As explained
Asset. The Special Mention Accounts are usually below.
categorized in terms of duration. For example, in
the case of SMA -1, the overdue period is between Option 4 is not correct: There is no provision of
31 to 60 days. On the other hand, an overdue presenting separate Gender Budget statement.
between 61 to 90 days will make an asset SMA -2. Since 2005-06, a note on Gender Budgeting is
In the case of SMA -NF, non-financial indications being issued in the form of Statement as a part
about stress of an asset is considered. of the Expenditure Budget Document by the
Ministry of Finance.
Statement 2 is correct: According to RBI, terms
loans on which interest or installment of principal The list of Budget documents presented to
remain overdue for a period of more than 90 days the Parliament, besides the Finance Minister's
from the end of a particular quarter is called a BudgetSpeech are:
Non-performing Asset. However, in terms of A. Annual Financial Statement (AFS)
Agriculture / Farm Loans, the NPA is defined as B. Demands for Grants (DG)
under: C. Finance Bill
For short duration crop agriculture loans such as D. Statements mandated under FRBM Act:
paddy, Jowar, Bajra etc. if the loan (installment /
(i) Macro-Economic Framework Statement
interest) is not paid for 2 crop seasons, it would be
termed as a NPA. For Long Duration Crops, the (ii) Fiscal Policy Strategy Statement
above would be 1 Crop season from the due date. (iii) Medium Term Fiscal Policy Statement

© Copyright:   www.nextias.com

https://t.me/TheHindu_Zone_official
https://t.me/IAS201819 https://t.me/PDF4Exams https://t.me/PDF4Exams

Modern History of India, Revision of Test-2 & Current Affairs_190903 | 7

E. Expenditure Budget 5. Umbrella Programme for Development


F. Receipts Budget of Minorities
G. Expenditure Profile 6. Umbrella Scheme for Development of
Backward Classes, Differently Abled and
H. Memorandum Explaining the Provisions in the
other Vulnerable Groups
Finance Bill
(ii) Core Scheme: Focus of CSSs should be
I. Budget at a Glance
on schemes that comprise the National
J. Outcome Budget Development Agenda where the Centre
26. (c) and States will work together in the spirit of
Team India .Swachh Bharat Mission, Green
Statement 1 is correct: A web-based Government
Revolution, PMKSY, PMAY, National Health
Integrated Financial Management Information
mIssion and National Education Mission are
System (GIFMIS), administered by Controller
some of the core schemes.
General of Accounts has been set up as an e-office
and e-governance initiatives in central Ministries (iii)Optional: The Schemes where States are free
and Departments to choose the ones they wish to implement.
Funds for these schemes would be allocated
Statement 2 is correct: It is set up for budgeting,
to States by the Ministry of Finance as a lump
accounting, expenditure and cash management for
sum.
more effective fiscal management of Government.
Additional information:
27. (a) Central Sector Schemes are entirely and directly
Option 1 and 2 are correct: Capital receipts may funded and executed by the central government.
be debt creating or non-debt creating. Examples The schemes are formulated by the Centre, based
of debt creating receipts are—Net borrowing by on subjects from the Union List.
government at home, loans received from foreign
governments, borrowing from RBI. Examples of 29. (d)
non-debt capital receipts are—Recovery of loans, Growth in insurance sector are assesed on two
proceeds from sale of public enterprises (i.e., parameters:
disinvestment), etc. These do not give rise to debt. (i) Insurance Penetration: It is defined as the
Option 3 is not correct: Recoveries of loans and ratio of premium underwritten in a given
advances comes under the non- debt receipt year to the Gross domestic product(GDP).
classification. (ii) Insurance Density: It is defined as the ratio of
28. (b) preimum underwitten in a given year to total
population.
Centrally Sponsored Schemes(CSS) are the
schemes by the centre where there is financial Additional Information:
participation by both the centre and states. the As per Economic Survey 2018, insurance
ratio of centre to state financial contribution may penetration in India increased to 3.49% in 2016-17
vary from 50:50, 60:40, 70:30, 75:25, 90:10. CSS are from 2.71% in 2001. However, India lags behind
created on areas that are covered under the State some of the emerging economies in Asia such as
List and implementation is made by State/UT Malaysia (4.77%), Thailand (5.42%) and China
Governments. These are divided on the basis of (4.77%). The country's life insurance penetration
financial involvement of the states into three parts: was 2.72% and general insurance penetration
(i) Core of the core Schemes: These are for social was 0.77%; this compares to a global insurance
protection and social inclusion should form the penetration of 3.47% for life and 2.81%.
core of core and be the first charge on available India's insurance density has increased to $59.7
funds for the National Development Agenda. from $11.5 in 2001 - with life insurance density of
Soem prominent schemes under this are: $46.5 and general insurance density of $13.2. This
1. National Social Assistance Progamme compares to global average insurance density of
$353 for life and $285.3 for non-life.
2. Mahatma Gandhi National Rural
Employment Guarantee Programme 30. (b)
3. Umbrella Scheme for Development of Statement 1 is not correct: In the situation of
Schedule Castes "double coincidence of wants", what a person
4. Umbrella Scheme for Development of desires to sell is exactly what the other wishes
Schedule Tribes to buy. e.g if a person wants to buy apples by
selling mats, the other person must be ready to

© Copyright:   www.nextias.com

https://t.me/TheHindu_Zone_official
https://t.me/IAS201819 https://t.me/PDF4Exams https://t.me/PDF4Exams

8 | CSE (Prelims) Test Series-2019


buy that mat in exchange of apples in an order to (ii) Public expenditure priorities – moving them
have the transaction take place. away from subsidies and administration
Statement 2 is correct: Since Barter System towards previously neglected fields with
goods are directly exchanged without the use high economic returns
of money, double coincidence of wants is an (iii) Tax reform – broadening the tax base and
essential feature. cutting marginal tax rates
31. (c) (iv) Financial liberalization – interest rates
should ideally be market-determined
Statement 1 is not correct: Quantitative easing
is considered an uncoventional monetary policy (v) Exchange rates – should be managed to
in which a central bank purchases government induce rapid growth in non-traditional
securities or other securities from the market exports
in order to lower interest rates and increase the (vi) Trade liberalization
money supply. Quantitative easing increases the (vii) Increasing foreign direct investment (FDI)
money supply by flooding financial institutions – by reducing barriers
with capital in an effort to promote increased (viii) Privatization – state enterprises should be
lending and liquidity. Quantitative easing is privatized
considered when short-term interest rates are at
(ix) Deregulation – abolition of regulations that
or approaching zero, and does not involve the
impede the entry of new firms or restrict
printing of new banknotes.
competition (except in the areas of safety,
Statement 2 is not correct: It generally causes environment and finance)
depreciation in the value of home country's
(x) Secure intellectual property rights (IPR) –
currency.
without excessive costs and available to
32. (d) the informal sector Reduced role for the
Statement 1 is correct: In the situation of liquidity state.
trap, the nominal interest rate is close or equal 34. (a)
to zero, making the opportunity cost of holding
The Financial Sector Assessment Program
money zero and even if the monetary authority
(FSAP) is a comprehensive and in-depth analysis
increases money supply to stimulate the
of a country’s financial sector.
economy, people hoard money. Consequently,
excess funds may not be converted into new Statement 1 is correct: FSAP assessments are the
investment. joint responsibility of the IMF and World Bank
in developing economies and emerging markets
Statement 2 is correct: A liquidity trap is usually
and of the IMF alone in advanced economies.
caused by, and in turn perpetuates, deflation.
The FSAP includes two major components:
When deflation is persistent and combined with
a financial stability assessment, which is the
an extremely low nominal interest rate, it creates
responsibility of the IMF, and a financial
a vicious cycle of output stagnation and further
development assessment, the responsibility of
expectations of deflation that lead to a higher
the World Bank.
real interest rate. Two prominent examples of
liquidity traps in history are the Great Depression Statement 2 is not correct: Its recommendations
in the United States during the 1930s and the long are not binding for the IMF members.
economic slump in Japan during the late 1990s. 35. (c)
33. (b) Statement 1 is correct: Monetized deficit, also
The Washington Consensus is a set of policy known as the ‘net reserve bank credit to the
recommendations said to be followed by a number government’, is that part of the government
of international monetary institutions that deficit which is financed solely by borrowing
included the International Monetary Fund, the from the RBI. Monetized deficit is the sum of the
World Bank, and the World Trade Organization net issuance of short-term treasury bills, dated
(WTO). The initial name “Washington Consenus” securities (that is, long-term borrowing from the
was derived by economist John Williamson. RBI) and rupee coins held exclusively by the RBI,
There were ten specific points within the net of Government’s deposits with the RBI.
Washington Consensus: Statement 2 is correct: Monetization of deficits,
(i) Fiscal discipline – strict criteria for limiting which increases the money supply, is inflationary
budget deficits if the rate of growth of money supply is greater

© Copyright:   www.nextias.com

https://t.me/TheHindu_Zone_official
https://t.me/IAS201819 https://t.me/PDF4Exams https://t.me/PDF4Exams

Modern History of India, Revision of Test-2 & Current Affairs_190903 | 9

than the rate of increase of the demand for that are produced within the economy (i.e.
cash balances arising from the growth of the domestically). Hence, it does not include changes
economy. Thus, monetized deficits are an in the price of imported goods.
important indicator of the inflationary impact of
the increase in government’s budgetary deficits. 38. (b)
Additional Information Statement 1 is not correct: Gender budgets are
not separate budgets for women. Since 2005-06, a
Monetized deficit is different from the Traditional note on Gender Budgeting is being issued in the
Budget deficit in two ways- form of Statement 20 as a part of the Expenditure
(i) Traditional Budget deficit includes 91-day Budget Document Volume 1by the Ministry of
treasury bills held by both, the RBI and Finance. This GB Statement comprises two parts:
non-RBI entities whereas monetized deficit (i) Part A reflects Women Specific Schemes, i.e.
includes 91-day Treasury Bills held only by those which have 100% allocation for women.
the RBI.
(ii) Part B reflects Pro Women Schemes, i.e. those
(ii) Traditional Budget deficit includes only where at least 30% of the allocation is for
short-term sources of finance whereas women.
Monetized deficit includes long-term
securities also. Statement 2 is correct: The Gender Budgeting
Cells are envisaged to serve as focal points for
36. (d) coordinating gender budgeting initiatives within
Statement 1 is not correct: A cess may be in the their Ministries and across Departments.
nature of a tax or a fee but it is imposed for a specific Additional Information:
purpose, as identified in the charging legislation. Gender Budgeting is a powerful tool for
Article 270 of the Constitution describes a cess: A achieving gender mainstreaming so as to ensure
tax is a compulsory contribution collected by the that benefits of development reach women as
government from the public at large and is to be much as men. It is not an accounting exercise
used for a public purpose. On the other hand, a but an ongoing process of keeping a gender
fee is imposed by the government for a specific perspective in policy/ programme formulation,
facility or service being provided or rendered. its implementation and review. The rationale
Cess may be levied by the Union or state for gender budgeting arises from recognition
governments. Cess is named after the identified of the fact that national budgets impact men
purpose; the purpose itself must be certain and and women differently through the pattern of
for public good. In the case of a cess which has the resource allocation. Women, constitute 48% of
attributes of a tax (cess tax), while the taxpayer India’s population, but they lag behind men on
does not have the right to ask for a reciprocal many social indicators like health, education,
benefit, the proceeds ought to be spent only for economic opportunities, etc. Hence, they warrant
the earmarked purpose. In simple words, a cess special attention due to their vulnerability and
tax is an earmarked tax. lack of access to resources. The way Government
A surcharge is a tax on tax imposed for the budgets allocate resources, has the potential to
purposes of the union. A surcharge is dealt with transform these gender inequalities
under Article 271 of the Constitution. 39. (a)
Statement 2 is not correct: The proceeds The IIP is a composite indicator that measures
collected from a surcharge and a cess levied by the changes in the volume of production of a
the Union form part of the Consolidated Fund basket of industrial products during a given
of India. The funds need not be shared with the period with respect to the volume of production
State governments and are thus at the exclusive in a chosen base period. The data refers to the
disposal of the Union government. General Index as well as for each industry group.
37. (d) The IIP is compiled as a simple weighted arithmetic
mean of production relatives by using Laspeyre’s
Statement 1 is not correct: The Consumer Price
formula.The base year for the current series of IIP
Index measures the price level of all goods and
is 2011-12.
services that are bought by consumers within the
economy. Hence,it does not account for changes Statement 1 is not correct: It measures the short-
in the price of exported goods. term changes in the volume of production of a
basket of industrial products during a given period
Statement 2 is not correct: The GDP deflator
with respect to that in a chosen base period.
measures the price level of all goods and services

© Copyright:   www.nextias.com

https://t.me/TheHindu_Zone_official
https://t.me/IAS201819 https://t.me/PDF4Exams https://t.me/PDF4Exams

10 | CSE (Prelims) Test Series-2019


Statement 2 is correct: The IIP index is computed carried out in Rajasthan on May 18, 1974, which
and published by the Central Statistical was during the fourth 5 year plan (1969-1974).
Organisation (CSO) on a monthly basis. Statement 3 is not correct: During fifth 5 year
Additional Information: plan (1974-78), the Indian national highway
IIP covers 407 item groups. Sector wise, the system was introduced and many roads were
items included falls into 3 categories viz. widened to accommodate the increasing traffic.
Manufacturing (405 items), Mining (1 items) & 42. (d)
Electricity (1 item).
Statement 1 is not correct: Provisioning
40. (d) Coverage Ratio (PCR) is essentially the ratio of
All options are correct as explained below. provisioning to gross non-performing assets
and indicates the extent of funds a bank has
Composite Water Management Index: It has
kept aside to cover loan losses. From a macro-
been developed by NITI Aayog to assess and
prudential perspective, banks should build up
improve performance in efficient management
provisioning and capital buffers in good times
of water resources. The index comprises 9 broad
i.e. when the profits are good, which can be used
sectors with 28 different indicators covering
for absorbing losses in a downturn. This will
various aspects of ground water, restoration of
enhance the soundness of individual banks, as
water bodies, irrigation, farm practices, drinking
also the stability of the financial sector. It was,
water, policy and governance.
therefore, decided that banks should augment
School Education Quality Index: The SEQI their provisioning cushions consisting of specific
is a composite index that reports annual provisions against NPAs as well as floating
improvements of States on key domains of provisions, and ensure that their total PCR,
education quality, conceptualized and designed including floating provisions, is not less than 70
by NITI Aayog and the MHRD. The SEQI is per cent
divided into two categories: Outcomes and
Statement 2 is not correct: A higher provision
Governance & Management. These are further
coverage ratio means the bank is protecting itself
divided into three domains of Outcomes
better against its bad loans. A decline in the ratio
(Learning, Access and Equity) and two domains
means that provisions have not been made to the
of Governance & Management (Governance
extent of the rise in bad loans.
Processes and Structural Reforms).
Performance on Health Outcomes index: 43. (d)
“Healthy States, Progressive India' report: The Priority Sector means those sectors which the
report has been developed by NITI Aayog, with Government of India and Reserve Bank of India
technical assistance from the World Bank, and consider as important for the development
in consultation with the Ministry of Health and of the basic needs of the country and are to be
Family Welfare (MoHFW). The report ranks given priority over other sectors. The banks
states and Union territories innovatively on are mandated to encourage the growth of such
their year-on-year incremental change in health sectors with adequate and timely credit. Priority
outcomes, as well as, their overall performance Sector includes the following categories:
with respect to each other. (i) Agriculture
41. (a) (ii) Micro, Small and Medium Enterprises
Statement 1 is correct: 14 banks were nationalised (iii) Export Credit
by the government on July 19, 1969 during (iv) Education
fourth 5 year plan (1969-74). Commercial banks
(v) Housing
belonged to the private sector those times. Since
these commercial banks were run by business (vi) Social Infrastructure
houses, they failed in helping the government (vii) Renewable Energy
in many ways. Hence, the government decided (viii) Others
to nationalize 14 major commercial banks. Other Additional Information:
major events during fourth plan were: Green
Priority Sector Lending Certificates (PSLCs):
revolution, Indo-Pakistan war, Bangladesh
PSLCs are a mechanism to enable banks to
Liberation War.
achieve the priority sector lending target and
Statement 2 is not correct: Smiling Budhha sub-targets by purchase of these instruments in
underground nuclear test (Pokharan 1) was the event of shortfall.

© Copyright:   www.nextias.com

https://t.me/TheHindu_Zone_official
https://t.me/IAS201819 https://t.me/PDF4Exams https://t.me/PDF4Exams

Modern History of India, Revision of Test-2 & Current Affairs_190903 | 11

Targets for domestic scheduled commercial in pursuance of the Indradhanush action plan of
banks (excluding Regional Rural Banks and the Government and it is expected to strengthen
Small Finance Banks) and Foreign banks with 20 the banking sector and improve its efficiency and
branches and above: profitability.
Total priority sector lending: 40 per cent of Statement 2 is correct: The merger will lead to
Adjusted Net Bank Credit(ANBC) or Credit better management of high value credit exposures
Equivalent Amount of Off-Balance Sheet through focused monitoring and control over
Exposure, whichever is higher. (Sub target- cash flows instead of separate monitoring by six
Agriculture: 18 %, Micro Enterprises: 7.5%, different banks.
Advances to Weaker Sections: 10%) Additional Information:
Foreign banks with less than 20 branches: 40 % Indradhanush Action Plan: Under the
of ANBC or Credit Equivalent Amount of Off- Indradhanush Action Plan, the Government of
Balance Sheet Exposure, whichever is higher, to India proposed to make available
be achieved in a phased manner by 2020.
Rs .70,000 crores for recapitalization of public
44. (d) sector banks out of budgetary allocations for
Statement 1 is correct: With a view to bringing four years.
the identity closer to the cultural heritage of the Government has also allowed all PSBs to raise
country as also for showcasing her scientific capital from Public markets through Follow-on
advances, a new series of notes in a new design is Public Offer (FPO) or
being launched. The new design bank notes are Qualified Institutional Placement (QIP) by
distinctly different from the current Mahatma diluting Government of India holding upto
Gandhi Series of bank notes in colour, size and 52% in phased manner based on their capital
theme. The theme of the new series notes is requirement, their stock performance, liquidity,
India’s heritage sites. market conditions etc.
Statement 2 is correct: A few new other elements
46. (d)
added in these notes are numerals in devnagari
and the logo of Swachh Bharat. The new notes Statement 1 is correct: Fugitive Economic
also have design elements in myriad and intricate Offenders Act, 2018 lays down the measures
forms and shapes. to empower Indian authorities to attach and
confiscate proceeds of crime associated with
Statement 3 is correct: Number panels of the
economic offenders and the properties of the
notes are printed in fluorescent ink. The notes
economic offenders.
also have optical fibres.
Statement 2 is correct: The confiscation order of the
Both can be seen when the notes are exposed to
Special Court shall, to the extent possible, identify
ultra-violet lamp.
the properties in India or abroad that constitute
Additional Information: proceeds of crime which are to be confiscated
The other security features in new bank notes and in case such properties cannot be identified,
are: water mark, security thread, latent image of quantify the value of the proceeds of crime.
denomination numeral, denomination numeral Statement 3 is correct: Fugitive Economic Offenders
in colour shifting ink, number panels, see Act, 2018 also provides for disentitlement of the
through register, electro-type, bleed lines etc. fugitive economic offender from defending any
Heritage themes of various notes: civil claim.
Note denomination Heritage site/Motif Additional Information:
10 Motif of Sun Temple, Konark. A Fugitive Economic Offender is a person against
50 Motif of Hampi with Chariot. whom an arrest warrant has been issued in respect
of a scheduled offence and who has left India
200 Motif of Sanchi Stupa.
so as to avoid criminal prosecution, or being
500 Motif of Red Fort. abroad, refuses to return to India to face criminal
2000 Motif of the Mangalyaan. prosecution.
45. (c) The Fugitive Economic Offenders Act, 2018, inter
alia provides for–
Statement 1 is correct: The acquisition of
subsidiary banks of State Bank is an important (i) Making an application before the Special
step towards strengthening the banking sector Court for a declaration that an individual is a
through consolidation of public sector banks. It is fugitive economic offender.

© Copyright:   www.nextias.com

https://t.me/TheHindu_Zone_official
https://t.me/IAS201819 https://t.me/PDF4Exams https://t.me/PDF4Exams

12 | CSE (Prelims) Test Series-2019


(ii) Attachment of the property of a fugitive by the elaborate securities market regulations of
economic offender and proceeds of crime. a country, the legal status of ICO ‘tokens’ is up
(iii) Issue of a notice by the Special Court to the in air because many countries, including India,
individual alleged to be a fugitive economic haven’t yet framed any regulations for them.
offender. Companies that raise money through IPOs are
(iv) Confiscation of the property of an individual required to file and get approval for a detailed
declared as a fugitive economic offender or prospectus from regulators and provide ongoing
even the proceeds of crime. disclosures to investors. ICOs, however, skirt
all these rules and simply issue a white paper
(v) Appointment of an Administrator to manage
sketching out business plans.
and dispose of the confiscated property
under the Act. 49. (c)
47. (d) Statement 1 is not correct: All the members of
United Nations are not members of International
All the given options are correct.
Monetary Fund (IMF). The IMF is an organization
Department of Industrial Policy and Promotion of 189 countries and United Nations is of 193
(DIPP), the nodal department for 'Make in India', member countries.
would spearhead the initiative for the Champion
Statement 2 is not correct: Quota subscriptions
Sectors in manufacturing and Department of
are a central component of the IMF’s financial
Commerce would coordinate the proposed
resources. Each member country of the IMF is
initiative for the Champion Sectors in Services.
assigned a quota, based broadly on its relative
Accordingly, Department of Commerce, with
position in the world economy and there voting
wide stakeholder consultation coordinated the
powers depends on there quotas.
preparation of draft initial sectoral reform plans
for several services sectors and, subsequently the Statement 3 is correct: The IMF supports
action plan. These 12 sectors include: its membership by providing policy advice
to governments and central banks based on
1. Information Technology & Information
analysis of economic trends and cross-country
Technology enabled Services (IT & ITeS)
experiences; loans to help countries overcome
2. Tourism and Hospitality Services economic difficulties; concessional loans to help
3. Medical Value Travel fight poverty in developing countries.
4. Transport and Logistics Services
50. (c)
5. Accounting and Finance Services
Statement 1 is not correct: With a view to
6. Audio Visual Services improve the Governance of Public Sector Banks
7. Legal Services (PSBs), the Government decided to set up an
8. Communication Services autonomous Banks Board Bureau. The Bureau
9. Construction and Related Engineering will recommend for selection of heads - Public
Services Sector Banks and Financial Institutions and
help Banks in developing strategies and capital
10. Environmental Services
raising plans.
11. Financial Services
Statement 2 is correct: The Bureau recommends
12. Education Services for selection of heads of Public Sector Banks
48. (a) and Financial Institutions and help Banks in
developing strategies and capital raising plans.
Statement 1 is correct: ICOs are known as token
sales, which raise money from public investors to Statement 3 is correct: The Board have three ex-
bankroll a start-up. officio members and three expert members in
addition to Chairman. All the
IPO (Initial Public Offering) is the traditional way
by which companies go public, offering shares to Members and Chairman will be part time.
the financial institutions and the common man. 51. (c)
But instead of the shares you acquire in an IPO,
Statement 1 is correct: It is a common practice that
ICOs allot digital ‘tokens’ which represent an
a person makes a list of several activities before
informal ownership share in the business they
adopting a particular one to persue his/her goal.
fund.
Similarly, in production a producer leaves some
Statement 2 is not correct: While rights in the case alternatives before finally choosing to produce
of shares bought in an IPO are legally protected the output. So, while finally choosing one, the

© Copyright:   www.nextias.com

https://t.me/TheHindu_Zone_official
https://t.me/IAS201819 https://t.me/PDF4Exams https://t.me/PDF4Exams

Modern History of India, Revision of Test-2 & Current Affairs_190903 | 13

producer did forego the alternative production. limit as the per capita income increases, the
Let us take example of a farmer. He can produce growth starts declining. If the per capita income
either rice or wheat on a piece of land. If he is increased above the specific level through
has decided to produce wheat on this piece of saving and investment, it increases a growth in
land, he has to forego the produciton of rice population. The increase in population growth
for producing wheat. So, value of rice foregone as a result pushes down per capita income to its
(next best alternative) is the opportunity cost of stable level of equilibrium. Thus, the economy is
producing wheat.
caught in a low level equilibrium trap. To come
Statement 2 is correct: The concept of opportunity from this trap, the rate of increase of growth of
cost is applicable to the individual as well as income must be higher than the rate of increase
the society. The concept is very important
in population.
and is widely used in economics. Because
of its importance in economics, sometimes, Conditions conducive to trapping are detailed
opportunity cost is also called the economic cost. below:
1. A high correlation between the level of per
52. (d)
capita income and rate of population growth
Perfect competition was theorised as a market
structure where both consumers and firms were 2. A low propensity to direct additional per capita
price takers. income to increase per capita investment;
Feature of Perfect Competition: 3. Scarcity of uncultivated arable land
(i) Large number of sellers and buyers – As 4. Cultural inertia and economic inertia.
against monopoly market, a competitive 5. Inefficient production methods
market has large number of sellers selling Thus all the statements are the correct.
the commodity to a large number of buyers.
(ii) Homogeneous product: Under perfect 54. (b)
competition only a single product is sold. The Indian National Trust for Art and Cultural
This means all the sellers sell the same type Heritage (INTACH) was founded in 1984 in
of product to buyers. So the product is a New Delhi with the vision to spearhead heritage
perfect substitute. awareness and conservation in India. Today
(iii) Free entry and exit: Under perfect INTACH is recognized as one of the world’s
competition there is no bar on any new largest heritage organizations, with over 190
firm or producer to enter the market to sell Chapters across the Country.In the past 31 years
orproduce the product. Similarly if any INTACH has pioneered the conservation and
existing seller wants to exit then he is free to
preservation of not just our natural and built
do so.
heritage but intangible heritage as well.
(iv) Every seller wants to earn maximum profit.
Statement 1 is not correct: For its conservation
(v) The government’s role is to provide efforts, it got special consultative status with
protection to sellers and do not interfere in
UNESCO in 2007.
business.
Statement 2 is correct: It is a non-profit charitable
(vi) Knowledge about the product: Under
perfect competition sellers and buyers have organisation registered under the Societies'
perfect knowledge about the product. Registration Act, 1860. It aims to protect and
preserve India’s living, built, and natural heritage
(vii) There is no bar on factors of production such
as labour etc. to move from one production by undertaking necessary actions and measures.
unit to another to do work. INTACH’s mission to conserve heritage is
based on the belief that living in harmony with
Thus all the statements are correct.
heritage enhances the quality of life, and it is the
53. (b) duty of every citizen of India as laid down in the
The theory of Low Level Equilibrium Trap Constitution of India.
has been developed by R.R. Nelson for Additional Information
underdeveloped countries. It states that when
The INTACH Logo is based on the
per capita income increases above the minimum
anthropomorphic copper figure from Shahabad,
specific level, population tends to increase. But
when the growth rate reaches an upper physical Uttar Pradesh, belonging to the enigmatic
Copper Hoards of the Ganga Valley.

© Copyright:   www.nextias.com

https://t.me/TheHindu_Zone_official
https://t.me/IAS201819 https://t.me/PDF4Exams https://t.me/PDF4Exams

14 | CSE (Prelims) Test Series-2019


55. (a) 57. (b)
Statement 1 is correct: The Bay of Bengal Statement 1 is not correct: In foreign exchange
Initiative for Multi-Sectoral Technical and market, depreciation of currency is a situation
Economic Cooperation (BIMSTEC) is a regional when domestic currency loses its value in front of
organization comprising seven Member States a foreign currency if it is market-driven. It means
lying in the littoral and adjacent areas of the Bay depreciation in a currency can only take place if
of Bengal constituting a contiguous regional unity. the economy follows the floating exchange rate
This sub-regional organization came into being system. It makes exports more competitve in the
on 6 June 1997 through the Bangkok Declaration. global market.
It constitutes seven Member States: Bangladesh, Statement 2 is correct: Currency devaluations
Bhutan, India, Nepal, Sri Lanka, Myanmar and can be used by countries to achieve economic
Thailand policy. Having a weaker currency relative to the
Statement 2 is not correct: Initially, the economic rest of the world can help boost exports, shrink
bloc was formed with four Member States with the trade deficits and reduce the cost of interest
acronym ‘BIST-EC’ (Bangladesh, India, Sri Lanka payments on its outstanding government debts
and Thailand Economic Cooperation). During a
special Ministerial Meeting in Bangkok in Dec,1997 58. (c)
BAngladesh got included. With the admission of Statement 1 is correct: ICOMOS was founded
Nepal and Bhutan at the 6th Ministerial Meeting in 1965 in Warsaw as a result of the Venice
in y 2004, the name of the grouping was changed Charter of 1964.It works for the conservation
to BIMSTEC. and protection of cultural heritage places. It is
Statement 3 is not Correct: The Asian Development the only global non-government organisation
Bank is BIMSTEC’s development partner for of this kind, which is dedicated to promoting
improving transport infrastructure. it has provided the application of theory, methodology, and
support to prepare the Master Plan and task scientific techniques to the conservation of the
the BIMSTEC Transport Connectivity Working architectural and archaeological heritage across
Group (BTCWG) to work out the modalities for its the world.
implementation, giving due attention to the special Statement 2 is correct: ICOMOS is an advisory
circumstances and needs of the Member States. body to the World Heritage Committee of
UNESCO. Its role is to support the implementation
56. (a) of the cultural side of the Convention. Through
PARIVESH (Pro-Active and Responsive facilitation its global network composed of numerous
by Interactive, Virtuous and Environmental Single- committees and experts from various
window Hub) portal has been launched by the backgrounds, ICOMOS has been involved in the
government on the occasion of World Biofuel Day theoretical and practical implementation of the
(10th August). Convention for almost fifty years.
Statement 1 is correct: PARIVESH is a Single- 59. (a)
Window Integrated Environmental Management
System, developed in pursuance of the spirit of Project Navlekha is an initiative of Google
‘Digital India. It automates the entire process of through which offline content can be made
submitting the application and tracking the status fully editable and publish online without expert
of such proposals at each stage of processing. The digital knowledge. With project Navlekha,
main highlights of the portal are: Google plans to enable Indian publishers of
regional languages to take their content online.
(i) single registration and single sign-in for all This initiative will help Indian writers gain more
types of clearances (i.e. Environment, Forest, readership and recognition without having to
Wildlife and CRZ), struggle to sell their written copies physically. At
(ii) unique-ID for all types of clearances required present when one search in Indian languages, the
for a particular project and content available is just 1% of what’s available in
(iii) single Window interface for the proponent English.
to submit applications for getting all types of
60. (c)
clearances (i.e. Environment, Forests, Wildlife
and CRZ clearances). Scrub typhus, also known as bush typhus, is a
disease with symptoms similar to any viral
Statement 2 is not correct: It is an initiative of
fever. However it is caused by a bacteria called
Ministry of Environment, Forest and Climate
Orientia tsutsugamushi. Scrub typhus is spread
Change.

© Copyright:   www.nextias.com

https://t.me/TheHindu_Zone_official
https://t.me/IAS201819 https://t.me/PDF4Exams https://t.me/PDF4Exams

Modern History of India, Revision of Test-2 & Current Affairs_190903 | 15

to people through bites of infected chiggers (iv) It provides for viability gap funding scheme
(larval mites). The most common symptoms of for 2G ethanol Bio refineries of Rs.5000
scrub typhus include fever, headache, body aches, crore in 6 years in addition to additional
and sometimes rash. Most cases of scrub typhus tax incentives, higher purchase price as
occur in rural areas of Southeast Asia, Indonesia, compared to 1G biofuels.
China, Japan, India, and northern Australia. (v) It encourages setting up of supply chain
Anyone living in or traveling to areas where scrub mechanisms for biodiesel production from
typhus is found could get infected. non-edible oilseeds, Used Cooking Oil, short
61. (a) gestation crops.
NITI Aayog has launched “Pitch to MOVE” - a Expected benefits of the policy are: Reducing Import
mobility pitch competition that aims to provide Dependency,Cleaner Environment, Health benefits,
budding entrepreneurs of India a unique Municipal Solid waste management,Employment
opportunity to pitch their business ideas to a Generation,Additional Income to Farmers.
distinguished jury. Startups working in the various
63. (b)
fields of mobility can pitch their ideas to industry
leaders and Venture Capitalists for raising Millet is a common term to categorize small-
investments. It aims to identify and incentivise the seeded grasses that are often termed nutri-
startups, which will help the Government realize cereals or dryland-cereals, and includes
its vision of Shared, Connected, Intermodal and sorghum, pearl millet, ragi, small millet, foxtail
Environment Friendly Mobility for India. The millet, proso millet, barnyard millet, kodo
objective is to harness the latest disruption for millet and other millets. An important staple
generating employment and growth in our country cereal crop for millions of small holder dryland
farmers across sub-saharan Africa and Asia,
62. (a) millets offer nutrition, resilience, income and
On the occasion of World Biofuel day, 2018, livelihood for farmers even in difficult times.
“National Policy on Biofuels 2018” was unveiled They have multiple untapped uses such as food,
by the government. feed, fodder, biofuels and brewing. Millets are
Statement 1 is not correct: It envisages a target of nutritionally superior to wheat & rice owing to
20% blending of ethanol in petrol by 2030. their higher levels of protein with more balanced
Statement 2 is correct: It is expected that addition amino acid profile, crude fiber & minerals such
of 2G (second generation) bio refineries across the as Iron, Zinc, and Phosphorous.
Country will spur infrastructural investment in the All statements are correct:
rural areas. Adapted to low or no purchased inputs and
Additional Information: to harsh environment of the semi-arid tropics,
Salient features of the policy on Biofuels: millets are the backbone for dry land agriculture.
(i) Categorisation of biofuels as "Basic Biofuels" Photo-insensitive & resilient to climate change,
viz. First Generation (1G) bioethanol & biodiesel millets are hardy, resilient crops that have a
and "Advanced Biofuels" - Second Generation low carbon and water footprint, can withstand
(2G) ethanol, Municipal Solid Waste (MSW) to high temperatures and grow on poor soils with
drop-in fuels, Third Generation (3G) biofuels, little or no external inputs. In times of climate
bio-CNG etc. to enable extension of appropriate change they are often the last crop standing and,
financial and fiscal incentives under each thus, are a good risk management strategy for
category. resource-poor marginal farmers.
(ii) Expanding the scope of raw material for ethanol The government has increased the MSP of millets
production by allowing use of Sugarcane Juice, by more than 50 per cent of cost of production
Sugar containing materials like Sugar Beet, which is an important component of efforts to
Sweet Sorghum, Starch containing materials achieve the national commitment of doubling
like Corn, Cassava, Damaged food grains like farmers’ income by 2022.
wheat, broken rice, Rotten Potatoes, unfit for Additional Information
human consumption for ethanol production.
India is celebrating 2018 as the National Year
(iii) The Policy allows use of surplus food grains for of Millets and is promoting cultivation by
production of ethanol for blending with petrol
amending cropping pattern of areas which are
to realize better price rumeneration for farmers.
especially susceptible to climate change

© Copyright:   www.nextias.com

https://t.me/TheHindu_Zone_official
https://t.me/IAS201819 https://t.me/PDF4Exams https://t.me/PDF4Exams

16 | CSE (Prelims) Test Series-2019


64. (c) Indian Council of Philosophical Research is a
Statement 1 is correct: The Transiting Exoplanet Member Societies to FISP. The Council will be
Survey Satellite (TESS) will discover thousands sending delegation for participation in a -
of exoplanets in orbit around the brightest stars 1. Round Table on Buddhist Paradigm for New
in the sky. World Order
Statement 2 is correct: This first-ever spaceborne 2. Panel Discussion on Integral Humanism in
all-sky transit survey will identify planets India Thought
ranging from Earth-sized to gas giants, around a 3. Symposium on Gandhian Thought: Tradition
wide range of stellar types and orbital distances. and Modernity
No ground-based survey can achieve this feat.
Statement 1 is not correct: 22nd Congress was
Statement 3 is correct: TESS will create a catalog held on South Korea which was held before the
of thousands of exoplanet candidates using the recent event.
transit photometry method. After this list has
Statement 2 is not correct: India is being
been compiled, the TESS mission will conduct
represented by Indian Council of Philosophical
ground-based follow-up observations to
Research
confirm that the exoplanets candidates are true
exoplanets and not false positives. 67. (d)
Additional Information Statement 1 is not correct: Global Liveability
TESS will monitor the brightnesses of more Index is released by The Economist Intelligence
than 200,000 stars during a two year mission, Unit.
searching for temporary drops in brightness Statement 2 is not correct: The index assigns
caused by planetary transits. Transits occur cities scores on five broad parameters — stability,
when a planet's orbit carries it directly in front of healthcare, culture/environment, education, and
its parent star as viewed from Earth. infrastructure
65. (b) 68. (d)
Statement 1 is not correct: The Food Safety BNHS-India, a pan-India wildlife research
and Standards Authority of India (FSSAI) organization, has been promoting the cause
launched RUCO (Repurpose Used Cooking of nature conservation for the past 133 years,
Oil), an initiative that will enable collection and since 1883.BNHS Mission: Conservation of
conversion of used cooking oil to bio-diesel Nature, primarily Biological Diversity through
Statement 2 is correct: Total Polar action based on Research, Education and Public
Compounds(TPC) are formed on repeated frying. Awareness
The toxicity of these compounds is associated Statement 1 is not correct: BNHS has been
with several diseases such as hypertension, designated as a Scientific and Industrial Research
atherosclerosis, Alzheimer’s disease, liver Organization (SIRO) by Department of Science
diseases. Therefore, it is essential to monitor the & Technology, Government of India and is the
quality of vegetable oils during frying Partner of BirdLife International in India.
Additional Information Statement 2 is not correct: Accenture Labs in
The FSSAI believes India has the potential to Bengaluru and the Bombay Natural History
recover 220 crore litres of used cooking oil for Society (BNHS) in Mumbai have developed an
the production of biodiesel by 2022 through a co- Internet of Things-based platform that identifies
ordinated action birds. The cloud-based software uses Artificial
FSSAI has laid down regulations to monitor the Intelligence (machine learning and computer
usage of used cooking oil. These regulations vision) to recognise species from digital
prescribe the limit for Total Polar Compounds photographs.
(TPC) to be a maximum 25%, beyond which the 69. (a)
cooking oil is unsafe for consumption. Statement 1 is correct: Union Finance Minster
66. (d) launched the mandatory use of Public Finance
The World Congress of Philosophy is organized Management System (PFMS) in October 2017
every five years by the International Federation for all Central Sector Schemes. This was launced
of Philosophical Societies (FISP) in collaboration as PFMS would ensure that the benefits of the
with one of its member societies. The 24th World various Government Schemes reach to the last
Congress is being held in Beijing, China mile.

© Copyright:   www.nextias.com

https://t.me/TheHindu_Zone_official
https://t.me/IAS201819 https://t.me/PDF4Exams https://t.me/PDF4Exams

Modern History of India, Revision of Test-2 & Current Affairs_190903 | 17

Statement 2 is not correct: It is integrated not The islands located north of 10° north latitude
only with the core banking solution of banks are known as Andaman Group of islands while
handling plan funds, but also with the State islands located south of 10° north latitude are
Treasuries for efficient and effective tracking of called Nicobar Group of islands.
fund flow to the lowest level of implementation Some of the important islands of Andaman and
for plan scheme of the Government. Nicobar Islands are Neil Island, Havelock Island,
70. (c) Cinque, Little Andaman, Diglipur (Ross and
Smith), etc.
Statement 1 is not correct: As per existing
contractual regime of Production Sharing Kiltan island is a coral island belonging to the
Contracts (PSCs), existing Contractors are Amindivi Subgroup of islands of the Union
not allowed to explore and exploit CBM or Territory of Lakshadweep in India.
other unconventional hydrocarbons in already 72. (b)
allotted licensed/leased area. Similarly, CBM
Under the Foreigners (Restricted Areas) Order,
Contractors are not allowed to exploit any other
1963, the following areas have been declared as
hydrocarbon except CBM. However the new
policy permits exploration and exploitation of `Restricted’ Areas :-
unconventional hydrocarbons such as Shale (i) Andaman & Nicobar Islands - Entire Union
oil/gas, Coal Bed Methane (CBM) etc. It will Territory
be carried out under the existing Production (ii) Sikkim - Part of the State
Sharing Contracts (PSCs), CBM contracts and Statement 1 is not correct: Restriced Areas
Nomination fields to encourage the existing are areas declared so due to internal security
Contractors in the licensed/leased area to unlock concerns.
the potential of unconventional hydrocarbons in
Statement 2 is correct: A foreigner is not normally
the existing acreages
allowed to visit a Protected / Restricted Area
Statement 2 is not correct: With the approval unless it is established to the satisfaction of the
of this policy, there will be complete shift from Government that there are extraordinary reasons
‘One hydrocarbon Resource Type’ to ‘Uniform to justify such a visit
Licensing Policy’ which is presently applicable
in Hydrocarbon Exploration & Licensing Policy 73. (a)
(HELP) and Discovered Small Field (DSF) Policy. Election Commission of India launches Mobile
Additional Information: App “cVIGIL” to enable citizens to report on
Other Benefits violation of election code of conduct.
It will enable the realization of prospective “cVIGIL" is a user-friendly and easy to operate
hydrocarbon reserves in the existing Contract Android application. It will be operational only
Areas which otherwise would remain unexplored where elections are announced.
and unexploited. The application will be made available for general
It may lead to new investment in Exploration and use by all, right from the forthcoming Assembly
Production (E&P) activities which may increase elections in the States of Chhattisgarh, Madhya
chances of discovering new hydrocarbon. This Pradesh, Mizoram and Rajasthan. The practical
may lead to increased domestic production use of the app during the four-state Assembly
thereof is expected. polls will also serve as a pilot initiative before it
is put to extensive use during the next Lok Sabha
It may spur new investment and give impetus
General Elections
to economic activities, additional employment
generation and thus benefitting various sections 74. (c)
of society. Statement 1 is correct: The Asia-Pacific
This will lead to induction of new, innovative Institute for Broadcasting Development (AIBD),
and cutting-edge technology and forging established in 1977 under the auspices of
new technological collaboration to exploit UNESCO, is a unique regional inter-governmental
unconventional hydrocarbons. organisation servicing countries of the United
Nations Economic and Social Commission for
71. (d)
Asia and the Pacific (UN-ESCAP) in the field of
The Union Territory of the Andaman and electronic media development. It is hosted by the
Nicobar Islands, is situated between 6° and 14° Government of Malaysia and the secretariat is
North Latitude and 92° and 94° East Longitude. located in Kuala Lumpur.

© Copyright:   www.nextias.com

https://t.me/TheHindu_Zone_official
https://t.me/IAS201819 https://t.me/PDF4Exams https://t.me/PDF4Exams

18 | CSE (Prelims) Test Series-2019


Statement 2 is correct: Full membership of the 77. (b)
AIBD is confined to sovereign states and they are Statement 1 is correct: Swachh Sarvekshan
invited to designate the broadcasting authority Grameen-2018” (SSG, 2018) through an
of the country to be the beneficiary. The AIBD independent survey agency, developes
currently has 26 Full Members (countries), ranking of all districts of India on the basis
represented by 34 organisations, and 67 of quantitative and qualitative sanitation
Affiliate Members (organisations) with a total (Swachhata) parameters. This ranking will be
membership of 101 representing 48 countries based on a comprehensive set of parameters
and regions and over 50 partners in Asia, Pacific, including district-level surveys of public places
Europe, Africa, Arab States and North America. like schools, Anganwadis, PHCs, Haat bazaars,
75. (c) Panchayat and citizen’s perception of Swachhata
and their recommendations for improvement of
Statement 1 is not correct: India has become the program and data from the SBM-G IMIS. As
the third Asian country after Japan and South part of the Sarvekshan, States and Districts will
Korea to get the Strategic Trade Authorisation-1 be ranked for their sanitation and cleanliness
(STA-1) status. India is the 37th country to be status.
designated the STA-1 status by the United States.
Statement 2 is correct: This is explained as below.
Statement 2 is correct: Traditionally, the U.S. has
placed only those countries in the STA-1 list who The specific objectives of the Swachh Sarvekshan
are members of the four export control regimes: Grameen-2018 are:
Missile Technology Control Regime (MTCR), 1. To undertake a ranking of States and Districts
Wassenaar Arrangement (WA), Australia Group on the basis of their performance attained
(AG) and the Nuclear Suppliers Group (NSG). on key quantitative and qualitative SBM-G
India is a member of three of the four multilateral parameters.
export regimes (MTCR, WA, AG). 2. Engage rural community in improvement of
Statement 3 is correct: It paved the way to expand their sanitation status through an intensive
cooperation in civil space, defence, and other and holistic IEC campaign.
high-technology sectors and the complementary 3. Compare performance within Districts
steps of the U.S. to realign India in U.S. export in a State and across Districts and States
control regulations. nationally on key SBM-G parameters.
76. (a) 4. Engage with select Gram Panchayats and
citizens in every district and solicit their
Statement 1 is correct: The WCO represents 182
feedback and recommendation on improving
Customs administrations across the globe that
the program implementation.
collectively process approximately 98% of world
trade. As the global centre of Customs expertise, All these objectives fosters spirit of cooperative
the WCO is the only international organization federalism.
with competence in Customs matters and can Statement 3 is not correct: The Ministry of
rightly call itself the voice of the international Drinking Water and Sanitation launched a third
Customs community. party verification survey report to take stock of
Statement 2 is not correct: The World Customs the progress already made by the Swachh Bharat
Organization (WCO), established in 1952 as the Mission in rural India. The Quality Council of
Customs Co-operation Council (CCC) is an India (QCI) has conducted a transparent third-
independent intergovernmental body whose party assessment of the present status of rural
mission is to enhance the effectiveness and sanitation in all States and UTs.
efficiency of Customs administrations. 78. (b)
Statement 3 is not correct: India has become the The terms “Additional Surveillance Measures
Vice-Chair (Regional Head) of the Asia Pacific (ASM)” and “Graded Surveillance Measure
Region of World Customs (GSM)” had been in news recently are concerned
Organisation (WCO) for a period of two years, with functioning of Securities and Exchange
from July, 2018 to June, 2020. The WCO has Board of India.
divided its Membership into six Additional information:
Regions. Each of the six Regions is represented Additional Surveillance Measures (ASM):
by a regionally elected Vice-Chairperson to the
In continuation to various surveillance measures
WCO Council.
already implemented, SEBI and Exchanges,

© Copyright:   www.nextias.com

https://t.me/TheHindu_Zone_official
https://t.me/IAS201819 https://t.me/PDF4Exams https://t.me/PDF4Exams

Modern History of India, Revision of Test-2 & Current Affairs_190903 | 19

pursuant to discussions in joint surveillance 80. (c)


meetings, have decided that along with the other Statement 1 is correct: Central Adoption
measures there shall be Additional Surveillance Resource Authority (CARA) is a statutory body
Measures (ASM) on securities with surveillance of Ministry of Women & Child Development,
concerns based on objective parameters viz. Statement 2 is correct: It functions as the nodal
Price variation, Volatility etc. In view thereof, body for adoption of Indian children and is
shortlisting of securities for placing in ASM is mandated to monitor and regulate in-country
based on an objective criteria covering following and inter- country adoptions.
parameters and shall be reviewed on bimonthly Statement 3 is correct: CARA is designated as
basis for the applicability of ASM: the Central Authority to deal with inter-country
1. High Low Variation adoptions in accordance with the provisions
2. Client Concentration of the Hague Convention on Inter-country
3. No. of Price Band Hits Adoption, 1993, ratified by Government of India
in 2003. CARA primarily deals with adoption of
4. Close to Close Price Variation orphan, abandoned and surrendered children
5. PE Ratio through its associated /recognised adoption
6. Market Capitalisation agencies.
Graded Surveillance Measure (GSM): 81. (a)
Securities and Exchange Board of India (SEBI) Statement 1 is correct: The Vikas Engine is the
and Exchanges in order to enhance market workhorse liquid rocket engine used to power
integrity and safeguard interest of the second stage of the ISRO’s trusted Polar
investors, have introduced Graded Surveillance Satellite Launch Vehicle (PSLV). It is also used
Measures (GSM) wherein certain identified in the second stage and also at the four strap
securities shall be subjected to enhanced on stages of Geosynchronous Launch Vehicle
monitoring and surveillance actions. (GSLV) and in the twin-engine core liquid stage
(L110) of GSLV Mk-III.
The main objective of these measures is to:
Statement 2 is not correct: Indian Space Resource
1. Alert and advice investors to be extra cautious
Organisation (ISRO) has improved the thrust of
while dealing in these securities.
the Vikas engine. The main beneficiary of the
2. Advice market participants to carry out high-thrust Vikas engine is said to be the heavy-
necessary due diligence while dealing in these lifting GSLV-Mark III launcher, which ISRO
securities expects will now put 4,000-kg satellites to space.
79. (c) 82. (b)
The RRB(Amendment) Act 2014 mandates that The Global Slavery Index provides a country
of the capital issued by a RRB, 50% shall be held by country ranking of the number of people in
by the central government, 15% by the concerned modern slavery, as well as an analysis of the
state government and 35% by the sponsor bank. actions governments are taking to respond, and
Additional information: the factors that make people vulnerable. The Walk
Free Foundation is a global organisation with a
The Act allows RRBs to raise their capital
mission to end modern slavery in our generation
from sources other than the central and state
by mobilising a global activist movement,
governments, and sponsor banks. In such a
generating the highest quality research, enlisting
case, the combined shareholding of the central business, and raising unprecedented levels of
government and the sponsor bank cannot be capital to drive change in those countries and
less than 51%. Additionally, if the shareholding industries bearing the greatest responsibility for
of the state government in the RRB is reduced modern slavery today.
below 15%, the central government would have
Additional Information
to consult the concerned state government.
Sponsor banks: The Act provides for RRBs to be What is modern slavery?
sponsored by banks. These sponsor banks are Modern slavery refers to situations where one
required to (i) subscribe to the share capital of person has taken away another person’s freedom
RRBs, (ii) train their personnel, and (iii) provide — their freedom to control their body, their
managerial and financial assistance for the first freedom to choose to refuse certain work or to
five years. stop working — so that they can be exploited.

© Copyright:   www.nextias.com

https://t.me/TheHindu_Zone_official
https://t.me/IAS201819 https://t.me/PDF4Exams https://t.me/PDF4Exams

20 | CSE (Prelims) Test Series-2019


Freedom is taken away by threats, violence, a scholarship offered by the Indian Society of
coercion, abuse of power and deception. The net Oriental Art founded in 1907.
result is that a person cannot refuse or leave the
situation. 85. (b)
Statement 1, 2 and 3 are correct: Sultan Tipu
83. (c) was an innovator. His desire to change with the
Statement 1 is correct: Capital Adequacy Ratio times was symbolised in the introduction of a
(CAR) is the ratio of a bank’s capital in relation new calendar, a new system of coinage, and new
to its risk weighted assets and current liabilities. scales of weights and measures.
The reason minimum capital adequacy ratios His personal library contained books on such
(CARs) are critical is to make sure that banks diverse subjects as religion, history, military
have enough cushion to absorb a reasonable science, medicine, and mathematics. He showed
amount of losses before they become insolvent a keen interest in the French Revolution. He
and consequently lose depositors’ funds. It is planted a “Tree of Liberty” at Sringapatam and
decided by central banks and bank regulators became a member of the Jacobin Club.
to prevent commercial banks from taking excess
He tried to do away with the custom of giving
leverage and becoming insolvent in the process.
jagirs, and thus increase state income. He also
Statement 2 is correct: Basel III is an made an attempt to reduce the hereditary
internationally agreed set of measures developed possessions of the poligars. However, his
by the Basel Committee on Banking Supervision land revenue was as high as that of other
in response to the financial crisis of 2007-09. contemporary rulers—it ranged up to 1 /3rd of
The measures aim to strengthen the regulation, the gross produce.
supervision and risk management of banks
Statement 4 is not correct: The most famous
which also includes guidelines on CAR.
Indian zij was the Zij-i Muhammad Shahi,
Statement 3 is not correct: Basel norms are given compiled at Jai Singh II of Amber's Jantar Mantar
by the Bank for International Settlements (BIS). observatories. It is notable for employing the use
The Basel Process refers to the way in which the of telescopic observations.
BIS promotes international cooperation among
monetary authorities and financial supervisory 86. (b)
officials. After the death of Aurangzeb in 1707, his 65-year
old son Bahadur Shah-I became emperor. Some
84. (c)
of the important Later Mughal Emperors and
The songs composed at that time by Rabindranath their regime are described below.
Tagore, Rajani Kanta Sen, Dwijendralal Ray,
1. Bahadur Shah (1707-1712) was learned,
Mukunda Das, Syed Abu Mohammed, and others
dignified, and able. He followed a policy
later became the moving spirit for nationalists of
of compromise and conciliation. He was
all hues, ‘terrorists, Gandhian or Communists’
groping towards a solution of the problems
and are still popular.
besetting the Empire before his death in 1712.
Statement 1 is correct: It was, perhaps, in the
2. Muhammad Shah (1719-1748) was weak-
cultural sphere that the impact of the Swadeshi
minded and frivolous and over- fond of a life
Movement was most marked. Rabindranath’s
of ease and luxury. He neglected the affairs of
Amar Sonar Bangla was written during this
state. He even shared in the bribes taken by
time.The Swadeshi influence could be seen in
his favourite courtiers. Nadir Shah entered
Bengali folk music popular among Hindu and
the Indian territory towards the end of 1738,
Muslim villagers (Palligeet and Jan Gàn) and it
during his reign. He was succeed by Ahmad
evoked collections of India fairy tales such as,
Shah (1748-54) and Alamgir-II (1754-59) after
Thakurmar Jhuli(Grandmother’s tales) written
him.
by Daksinaranjan Mitra Majumdar which
delights Bengai children to this day. 3. Shah Alam II ascended the throne in 1759.
He spent the initial years as an Emperor
Statement 2 is correct: In art, this was the
wandering from place to place far away from
period when Abanindranath Tagore broke
his capital. He was a man of some ability and
the domination of Victorian naturalism over
ample courage. In 1764, he joined Mir Qasim
Indian art and sought inspiration from the rich
of Bengal and Shuja-ud-Daula of Avadh in
indigenous traditions of Mughal, Rajput and
declaring war upon the English East India
Ajanta paintings. Nandalal Bose, who left a major
Company. Defeated by the British at the
imprint on Indian art, was the first recipient of

© Copyright:   www.nextias.com

https://t.me/TheHindu_Zone_official
https://t.me/IAS201819 https://t.me/PDF4Exams https://t.me/PDF4Exams

Modern History of India, Revision of Test-2 & Current Affairs_190903 | 21

Battle of Buxar, he lived for several years at 90. (c)


Allahabad as a pensioner of the East India Raja Ram Mohan Roy -- He played a pioneer
Company. role in socio-religious as well as political reform
87. (d) movement in India.Raja Ram Mohan Roy, 'the
father of Indian renaissance', the founder of
The actual dominating British rule in India 'Brahmo Samaj' and the man who tirelessly
started only after the Battle of Plassey 1757. fought against the social evils prevailing in the
Hence during the period of 1757 to 1947 several Indian society. The country is paying tribute
famines occurs. Some famous famines during to this great social reformer and educationalist
the British rule were Odisha famine of 1866–67, who also worked as the messenger for modern
Great Famine of 1876–78, and Bengal famine of education in India during the British period.
1943.
1. Abolition of Sati: He succeeded in
Statement 1 is correct: Forceful cultivation of persuading Lord William Bentick to abolish
Indigo, Cotton and various other cash crop eg. it in 1829. He advocated for various reform in
Jute, Opium to meet the demand of local British women’s right including inheritance widow
Industry. Hence diversion of land for cash crop, remarriage
lead to shortage of food crops.
2. He fought for the introduction of modern
Statement 2 is correct: During the era of colonial education and laid emphasis on rationality.
rule exports from India constantly increased. However he didn't favour Sankrit education
Indian raw material becomes the source for but a holistic education.
British Industrial Revolution. Food grains were
the major constituent of these exports. 3. Roy acted against the evil practices that
existed in the Hindu society, especially, the
Statement 3 is correct: During the Bengal practice of ‘Sati’, wherein a woman jumps
Famines, due to ineffective control of rise in the into the funeral pyre of her husband. Roy's
price of rice leads to increase in hoarding strategy. sister was a victim of 'Sati'. His Samaj raised
Policy measures like Essential Articles Restricted its voice against polygamy, child marriage,
Acquisition Order-1943 and abolishment of caste system, untouchability, superstitions
inter provincial trade barriers during the Bengal and use of intoxicants. Meanwhile, Roy
Famines leads to Hoarding and Black-Marketing encouraged inter-caste marriages, women's
of Food Grains. Hence Shortage of Food Supply education and widow re-marriages.
for normal people and high cost lead to Famines.
4. He defended Hinduism and its Vedanta
88. (a) Philosphy Also he was a firm believer in
The Caste Disabilities Removal Act, 1850, was internationalism/ universal humanism.
a law passed in British India under East India 91. (d)
Company rule, that abolished all laws affecting
the rights of persons converting to another Statement 1 is not correct: Lord Dalhousie
religion or caste. The new Act allowed Indians opened the first telegraph line between Calcutta
who converted from one religion to another and Agra in 1853.
religion equal rights under no law, especially in Statement 2 is not correct: In 1853, the first
the case of inheritance commercial passenger service in India when a
train left Bombay's Bori Bunder for Thane, with
89. (d) a 21-gun salute. A number of railway lines were
Statement 1 is not correct: Captain William there before that but all were after 1835.
Hawkins visited to the Jahangir’s court in
1609 to obtain permission to open a factory at 92. (d)
Surat especially western coast. Jahangir was Factories Act of 1881 was the first legislation
succeeding his father Akbar since his death in related to labor reform under the British India. It
1605. was passed under the pressure of British Cotton
Statement 2 is not correct: Later on, this Royal mill owners who were facing competition from
Farman was extended to open factories across Indian Mill’s due to cheap labor and lack of
the Mughal Empire in 1615 when Thomas Roe regulation. The British mill owner was mainly
visited to the Mughal Court. Hence Thomas Roe interested in increasing the cost of production in
got a Royal firman (decree) to open factory and Indian mills and hence supported the legislation.
trade in any part of the Mughal Empire As per the Act, Children below the age of 7 years
were to be employed in any factory

© Copyright:   www.nextias.com

https://t.me/TheHindu_Zone_official
https://t.me/IAS201819 https://t.me/PDF4Exams https://t.me/PDF4Exams

22 | CSE (Prelims) Test Series-2019


93. (b) Sambhalpur – 1849
Some of the important press legislations during Udaipur – 1852
British era were. Jhansi – 1853
1. Press Act of 1835 or Metcalfe Act Nagpur – 1854
Metcalfe (Governor General – 1835 – 36) repealed Annexation of Awadh: Awadh was annexed
the obnoxious 1823 ordinance according to which finally in February 1856 on the ground of
press without licence was a penal offence. Thus “misgovernment”. via a proclamation.The British
the Press Act 1835 was named, “liberator of the alleged that the Nawab continuously neglected
Indian press”. and the whole of Awadh had fallen into the
2. Licensing Act, 1857: This act imposed licensing constantly increasing confusion, violent disorders
restriction and the right to stop publication and and oppression.
circulation of book, newspaper or printed matter
reserved with the Government. 96. (b)
3. Vernacular Press Act, 1878: It was constituted Founders of the three autonomous kingdoms:
for ‘better control’ of the vernacular press and Hyderabad: Nizam-ul-mulk
effectively punished and repressed seditious Bengal: Murshid Quli Khan and Alivardi Khan
writing. Awadh: Saadat Khan and Safdar Jang
Statement 1 is not correct: The subject content of the They were men of high personal morality.Their
Press Act of 1835 was licensing of press and its salient lives give lie to the belief that all the leading nobles
feature were diluted by the Licensing Act 1857. of the 18th century led extravagant and luxurious
Statement 2 is correct: Vernacular Press Act, 1878 was lives. It was only in their public and political
constituted for ‘better control’ of the vernacular press dealings that they resorted to fraud, intrigue and
and effectively punished and repressed seditious treachery,
writing. The Vernacular Press Act excluded English-
language publications. 97. (d)

94. (b)
Satatement 1 and 2 are correct: Indian Renaissance
was a mix of two type of movements: Revivalistics
and Reformistic. Revivalistic Movements laid
emphasis on recovery of spiritual tradition and
rich heritage of India. Arya samaj was one of them.
Reformistic movements aimed at overhaling the
stagnant traditions. For spreading mass awareness,
various religious sects were formed.
Indians were also taking active role in the field
of sciences. Jagdish Chandra Bose pioneered the
investigation of radio and microwave optics, made
significant contributions to botony and laid the
foundations of experimental science in the Indian
subcontinent.
Statement 3 is not correct: JC Bose was not
comtemporary of Raja Ram Mohan Roy. He was
born two decades after the death of Raja Ram Mohan
Roy.
95. (d)
Dotrine of Lapse: The Doctrine of Lapse was an
annexation policy followed widely by Lord Dalhousie
when he was India’s Governor-General from 1848 to Chinsura is a located on Hoogly river in bengal.
1856.After Dalhousie became the Governor-General, Serampore is located south to chinsura in bengal
in 1848, Satara was acquired as per this doctrine. on Hoogly river.
Other major states annexed by the British as per the Salsetee is an island in Maharastra.
Doctrine of Lapse: Masulipatnam is in Andhra Pradesh. The
Jaipur – 1849 company established its first factoy of south
India in Masulipatnam.

© Copyright:   www.nextias.com

https://t.me/TheHindu_Zone_official
https://t.me/IAS201819 https://t.me/PDF4Exams https://t.me/PDF4Exams

Modern History of India, Revision of Test-2 & Current Affairs_190903 | 23

98. (c) Deccan Uprisings: In May and June 1875,


Gopal Hari Deshmukh (18 February 1823 – 9 peasants of Maharashtra in some parts of Pune,
October 1892) was an Indian activist, thinker, Satara and Ahmednagar districts revolted
social reformer and writer from Maharashtra. He against increasing agrarian distress. The Deccan
wrote against the evils of the caste system which Riots of 1875 targeted conditions of debt peonage
was strongly prevalent in India in his times, (kamiuti) to moneylenders. The rioters' specific
condemned harmful Hindu religious orthodoxy, purpose was to obtain and destroy the bonds,
and attacked the monopoly in religious matters decrees, and other documents in the possession
and rituals which Brahmin priests had controlled of the moneylenders. It was rooted in the ill
through a long tradition. effects of the Ryotwari system.
He took a leadership role in founding Gyan 100. (a)
Prakash, Indu Prakash, and Lokhitwadi Statement 1 is correct: The Pitts India Act, 1784
periodicals in Maharashtra. He established (The East India Company Act 1784) gave the
in Ahmedabad a branch of Prarthana Samaj, British Government supreme control over the
founded an institute promoting remarriages of Company’s affairs and its administration in
widows, and invigorated Gujarat Vernacular India. It established dual system of governance:
Society. Government by Board of Control and
99. (d) Government of Court of Directors.It resulted
in dual control or joint government in India by
Pabna Revolt: Pabna Peasant Uprising (1873-
British government and the British East India
76) was a resistance movement by the peasants
Company, with British government having the
("Ryots") against the landlords in Bengal
ultimate authority.
("zamindars") in Pabna.It was led by Keshab
Chandra Roy. So it was a revolt in zamindari Statement 2 is not correct: Regulating Act of
system of settlement. Zamindari system (also 1773, the first attempt towards Centralised
known as permanent settlement) was present in Administration.In this act, Governor of Bengal
bengal and orissa became Governor General for all British
territories in India.Under the Act,Bombay and
Indigo Revolt: Till the late nineteenth century,
Madras Presidency subordinated to Bengal
planters were reluctant to expand the area under
Presidency in certain matters.
nij cultivation. Less than 25 per cent of the land
producing indigo was under plantation system. Statement 3 is not correct:The First Anglo-
The rest was under an alternative mode of Maratha War (1775–1782) was the first of three
cultivation – the ryoti system. Under the ryoti Anglo-Maratha wars fought between the British
system, the planters forced the ryots to sign a East India Company and Maratha Empire in
contract, an agreement (satta). At times they India. The war began with the Treaty of Surat
pressurised the village headmen to sign the and ended with the Treaty of Salbai.
contract on behalf of the ryots. Those who signed The First Anglo–Mysore War (1766–1769) was a
the contract got cash advances from the planters conflict in India between the Sultanate of Mysore
at low rates of interest to produce indigo. and the East India Company.
nnnn

© Copyright:   www.nextias.com

https://t.me/TheHindu_Zone_official

Das könnte Ihnen auch gefallen